Comprehensive Review in Clinical Neurology: A Multiple Choice Question Book for the Wards and Boards

Chapter 9. Neuromuscular I (Neurophysiology, Plexopathy, and Neuropathy)

Question

Questions 1–2

1. A 3-year-old boy is seen by a pediatric neurologist for multiple concerns including cognitive and motor developmental delay. The parents reported that the patient did not walk like his siblings, but rather seemed to walk on the insides of his feet. Unlike his siblings, he also had tightly curled hair. Examination revealed nystagmus, bilateral weakness of the legs, and brisk reflexes with ankle clonus. One year later, the patient requires insertion of a peg tube, has developed significant spasticity in his arms and legs, and has had significant vision loss. He has significant language delay as well. Two years later, the patient passes away from complications of aspiration pneumonia, and on autopsy, pathologic analysis of nerve fibers demonstrates large focal axonal swellings filled with neurofilaments. What is the most likely diagnosis in this patient?

a. Dejerine-Sottas syndrome

b. Charcot-Marie-Tooth (CMT) type 2A

c. CMT type 4

d. Giant axonal neuropathy

e. Metachromatic leukodystrophy

2. A 25-year-old man is referred to the clinic by his ophthalmologist. In early childhood, he was noted to have a retinal disorder, and in his teenage years, he was denied a driver’s license due to night blindness and visual field deficits. In recent years, he had begun to complain of tingling in his legs and later, of clumsiness and weakness. Examination revealed evidence of bilateral symmetric sensory loss and distal bilateral lower extremity weakness. A serum phytanic acid level is obtained and is elevated. What is the most likely diagnosis in this patient?

a. Myoneurogastrointestinal encephalopathy

b. Kearns-Sayre syndrome

c. Neurogenic muscle weakness, ataxia, and retinitis pigmentosa syndrome

d. Abetalipoproteinemia

e. Refsum’s disease

Questions 3–4

3. Which of the following is incorrect regarding electrophysiologic studies of the peripheral nervous system?

a. SNAP amplitude is a measure of the number of axons that conduct between the stimulation and recording sites

b. Sensory distal latency is the time it takes for the action potential to travel between the nerve stimulation site and the recording site

c. Axon loss lesions result in reduced conduction velocities

d. CMAP amplitude depends on the status of the motor axons, neuromuscular junctions, and muscle fibers

e. The F-wave and H-reflex are late responses

4. Which of the following is incorrect regarding electrophysiologic studies of the peripheral nervous system?

a. CMAP amplitudes may be reduced in axon loss lesions

b. Prolonged distal latency is seen in demyelinating lesions

c. The H-reflex is the electrophysiologic equivalent of the ankle reflex

d. The H-reflex is obtained by stimulating the tibial nerve

e. The F-wave is obtained after submaximal stimulation of a motor nerve

5. Regarding needle EMG, which of the following is incorrect?

a. Insertional activity is increased in denervated muscles

b. Fibrillation and fasciculation potentials are examples of spontaneous activity

c. Short-duration motor unit potentials (MUPs) are seen more frequently in myopathic processes

d. Large polyphasic MUPs are seen in acute neuropathic lesions

e. Reduced recruitment is seen in axon loss lesions

6. Which of the following is incorrect regarding the evaluation of a radiculopathy associated with an axon loss intraspinal canal lesion?

a. Fibrillation potentials can be seen in a segmental myotome 3 weeks after the onset

b. There are abnormal sensory SNAPs in a segmental dermatome

c. Reinnervation or collateral innervation occurs in a proximal to distal gradient

d. The H-reflex tests the S1 reflex arc, and is helpful in the diagnosis of S1 radiculopathy

e. Large polyphasic motor unit potentials can be detected in chronic radiculopathies

Questions 7–8

7. A patient presents for evaluation of weakness in the upper extremities that had been occurring for the prior few months. NCS and EMG were unremarkable. With rapid repetitive stimulation and evaluation after exercise, there was no increase in CMAP amplitude. Slow repetitive nerve stimulation was obtained, and is shown in Figure 9.1. Which of the following is the most likely diagnosis?

FIGURE 9.1 Two-Hertz repetitive nerve stimulation (Courtesy of Dr. Robert Shields)

a. A myopathy

b. A demyelinating neuropathy

c. An axon loss neuropathy

d. Myasthenia gravis

e. Botulism

8. A patient is referred for an EMG/NCS for a possible diagnosis of a neuromuscular junction disorder. Which of the following is correct?

a. CMAP increment after rapid repetitive stimulation is a feature of myasthenia gravis (MG)

b. CMAP increment after brief exercise is a feature of MG

c. A decrement in the CMAP after 2- to 3-Hz repetitive stimulation is consistent with MG

d. Abnormal jitter on single-fiber EMG is a very specific finding for the diagnosis of MG

e. Sensory NCS are typically abnormal in MG

9. A patient with lung cancer is being referred for evaluation of Lambert-Eaton myasthenic syndrome (LEMS). Which of the following is incorrect?

a. Needle EMG is usually normal in LEMS

b. Low to borderline-low CMAP amplitudes at rest are common in LEMS

c. Slow repetitive stimulation (2 to 3 Hz) results in a decremental response of the CMAP amplitudes

d. Rapid repetitive stimulation (20 to 50 Hz) results in an incremental response of the CMAP amplitudes

e. Voluntary single-fiber jitter analysis helps to distinguish myasthenia gravis from LEMS

10. Regarding the types of skeletal muscle fibers, which of the following is correct?

a. Type I fibers have low oxygen consumption

b. Type IIa fibers are fast with large glycolytic capacity

c. Type I fibers are large in size

d. Type IIb fibers are fast with high oxidative capacity

e. Type IIb fibers are slow

Questions 11–12

11. A 55-year-old man presents with weakness, which initially developed in the lower extremities. He undergoes evaluation and treatment and is eventually admitted to a rehabilitation facility. He returns 9 weeks later with worsening of his weakness, which is affecting upper extremities distally and lower extremities proximally and distally. He also has distal paresthesias, and his examination demonstrates diffuse areflexia. CSF shows 3 WBCs/mm3 (normal up to 5 lymphocytes/mm3) and protein is 100 mg/dL (normal up to 45 mg/dL). An EMG/NCS is obtained and right median nerve NCS is shown in Figure 9.2. Which of the following is the most likely diagnosis?

FIGURE 9.2 Right median nerve motor NCS (Courtesy of Dr. Robert Shields)

a. Guillain-Barre syndrome

b. Chronic inflammatory demyelinating polyneuropathy

c. Multifocal motor neuropathy

d. Lambert-Eaton myasthenic syndrome

e. Myasthenia gravis

12. Which of the following is incorrect regarding this condition?

a. Sensory NCS are required to establish the diagnosis

b. There may be prolongation of distal motor latencies

c. CSF albuminocytologic dissociation can be seen in this condition

d. Sural nerve biopsy shows evidence of inflammation, demyelination, and remyelination

e. Conduction block is a common finding

13. Which of the following is incorrect regarding an axon loss peripheral nerve injury?

a. Fibrillation potentials appear by the third week of the injury

b. Conduction block 10 days after the injury suggests segmental demyelination

c. The presence of conduction block can help localize the site of segmental demyelination

d. NCS 3 weeks after the injury are useful to localize a focal axon loss lesion

e. Axon loss leads to wallerian degeneration

14. A 62-year-old retired secretary presents to the clinic complaining of painful tingling sensations and numbness in her left thumb, index, and middle finger that wake her up at night. On examination, motor power in her left arm, forearm, and hand is normal. There is subtle loss of pinprick and light touch on the distal first to third digits (at the finger tips). Biceps and brachioradialis deep tendon reflexes are normal. A tracing from her median nerve sensory NCS is shown in Figure 9.3 (normal median nerve SNAP latency is 4.0 ms); motor NCS and EMG are normal. What is the most likely diagnosis?

FIGURE 9.3 Left median nerve sensory NCS (Courtesy of Dr. Robert Shields)

a. C6 radiculopathy

b. C7 radiculopathy

c. Carpal tunnel syndrome

d. Median neuropathy at the elbow

e. Brachial plexopathy

Questions 15–16

15. A 28-year-old woman presents with numbness and tingling in her feet. She was seen in the emergency department, and after no neurologic abnormalities were found, she was reassured and discharged home. Four days later she returns, unable to walk. She complains of numbness and tingling from her toes up to just above her knees, and also affecting her hands. On examination, she has distal more than proximal weakness in the lower extremities and subtle weakness in her hands. Ankle and patellar reflexes are absent. She recalls having a viral illness a couple of weeks ago. Which of the following is the most likely diagnosis?

a. Guillain-Barre syndrome

b. Multiple sclerosis

c. Stroke

d. Myelopathy

e. Myasthenia gravis

16. The patient is admitted to the hospital, and continues to worsen. Which of the following tests should be obtained to ensure adequate care, and may affect the subsequent management of this patient?

a. MRI of the lumbar spine

b. Evaluation of respiratory parameters, including negative inspiratory force and vital capacity

c. MRI of the brain

d. CSF studies for various antibodies, oligoclonal bands, and myelin basic protein

e. EMG/NCS as soon as possible

17. Regarding innervation of the upper extremity, which of the following is incorrect?

a. The brachial plexus is formed from the anterior rami of the C5 to T1 nerve roots

b. The middle trunk is formed from the C7 root

c. The lower (inferior) trunk is formed from the C8 and T1 roots

d. The dorsal scapular nerve is the only nerve that branches directly off the nerve roots

e. The cords of the brachial plexus are named according to their anatomic relationship to the axillary artery

Questions 18–19

18. A 40-year-old man presents to the clinic complaining of bilateral lower extremity sharp burning pains. Examination reveals bilateral symmetric loss of sensation to temperature and pinprick, with mild reduction in vibratory sense and fine touch, and normal proprioception. Review of systems reveals dyspnea on exertion, early satiety, erectile dysfunction, and constipation. Routine laboratory testing and monoclonal protein analysis are negative. His father suffered from similar symptoms in middle age, and was in a wheelchair prior to his death at the age of 65 from heart and renal failure. What is the most likely diagnosis in this patient?

a. Familial amyloid polyneuropathy type 1

b. Familial amyloid polyneuropathy type 2

c. Familial amyloid polyneuropathy type 3

d. Primary amyloidosis

e. Secondary amyloidosis

19. A 55-year-old woman presents with complaints of tingling and sensory loss over the right thumb, second digit, and third digit that wake her up at night. She has a history of left carpal tunnel syndrome, requiring surgery 3 years earlier. Examination reveals bilateral symmetric distal loss of pinprick sensation in the feet, but it is otherwise normal. Her mother and sister both have had bilateral carpal tunnel release. Review of systems is otherwise negative. What is the most likely diagnosis in this patient?

a. Familial amyloid polyneuropathy type 1

b. Familial amyloid polyneuropathy type 2

c. Familial amyloid polyneuropathy type 3

d. Primary amyloidosis

e. Secondary amyloidosis

20. A patient with long-standing diabetes has autonomic dysfunction. Which of the following is least likely to occur with diabetic autonomic neuropathy?

a. Impotence

b. Silent myocardial infarction

c. Resting tachycardia and loss of respiratory influence on the heart rate

d. Diarrhea that occurs typically during the daytime

e. Delayed gastric emptying

Questions 21–22

21. A 13-year-old girl is brought to the clinic by her mother for bilateral foot drop. As an infant and young child, she had always been clumsy and seemed to trip on her own feet, and could not participate in ballet classes. Over more recent years, her feet became noticeably weak. On examination, she had bilateral symmetric weakness of foot dorsiflexion and plantarflexion. She had hammertoes and high-arched feet. Ankle and knee deep tendon reflexes were absent. Examination of the mother also revealed hammertoes and high-arched feet. Conduction velocities of approximately 30 cm/second were revealed in all NCS of the legs, with no identifiable conduction block. What is the most likely diagnosis in this patient?

a. Charcot-Marie-Tooth (CMT) 1

b. CMT2

c. A muscular dystrophy

d. CMT3

e. CMT4

22. Regarding the various manifestations of the different subtypes of Charcot-Marie-Tooth (CMT), which of the following statements is incorrect?

a. Hammertoes and spine deformities are more prominent in CMT1 than CMT2

b. CMT2 typically has a later age of onset as compared to CMT1

c. CMT3, or Dejerine-Sottas syndrome, presents in infancy and typically leads to disabling weakness

d. Respiratory muscle involvement is common in the most common type of CMT

e. In some subtypes of CMT4, one of the axonal CMTs, vision or hearing loss occurs

23. Which of the following is correct regarding the median nerve?

a. The median nerve arises solely as a continuation of the medial cord

b. The median nerve carries C6 to T1 fibers

c. The median nerve innervates all forearm flexors

d. The median nerve innervates all intrinsic hand muscles

e. The median nerve innervates only forearm and hand muscles; it does not innervate any upper arm muscles

24. Which of the following treatment options has evidence to support its use for Guillain-Barre syndrome?

a. Intravenous immunoglobulins (IVIGs) combined with steroids

b. Steroids alone

c. Plamapheresis combined with steroids

d. Pyridostigmine

e. IVIGs or plasmapheresis

Questions 25–26

25. Which of the following is incorrect regarding the lumbosacral plexus?

a. The femoral nerve forms from posterior divisions of L2, L3, and L4

b. The obturator nerve forms from anterior divisions of L2, L3, and L4

c. Iliohypogastric, ilioinguinal, and genitofemoral nerves arise from the lumbosacral trunk

d. The lumbosacral trunk joins the sacral plexus

e. The lateral femoral cutaneous nerve arises from L2 and L3

26. Which of the following is incorrect regarding the lumbosacral plexus?

a. The lumbosacral trunk originates from L4 and L5

b. The posterior femoral cutaneous nerve provides sensory innervation to the lower buttock and posterior thigh

c. The pudendal nerve arises from S2, S3, and S4 and provides sensory innervation to the perineal region

d. The tensor fascia latae abducts the thigh when the hip is extended

e. The largest nerve of the lumbosacral plexus is the sciatic nerve

27. A 22-year-old woman presents to the neuromuscular clinic with painless right foot drop. NCS show a conduction block at the fibular head. She admits to frequently crossing her legs. She is prescribed an ankle-foot orthosis, undergoes physical therapy, and does well, eventually regaining back all of the strength of her right foot. Six years later, she presents with weakness of her intrinsic hand muscles, and NCS show conduction block of the ulnar nerve at the elbow. She does recall hitting her “funny bone” several times a few days prior to evaluation. Her family history reveals that her father and brother had a history of similar episodes of weakness in the foot or arm since they were young adults. Which of the following is incorrect regarding the diagnosis of this patient?

a. The peroneal nerve is most often affected, followed by the ulnar nerve

b. It is caused by a deletion in peripheral myelin protein 22 (PMP22) gene

c. It is autosomal dominant in inheritance with incomplete penetrance

d. This patient has hereditary neuralgic amyotrophy

e. Neuropathologic analysis of a nerve biopsy from this patient would show sausage-like thickening of the myelin, known as tomacula

28. Which of the following is correct regarding lower extremity innervation?

a. Gluteus medius and tensor fascia latae receive innervation from the superior gluteal nerve

b. Gluteus minimus receives innervation from the inferior gluteal nerve

c. The long head of the biceps femoris is innervated by the peroneal division of the sciatic nerve

d. Tibialis anterior is innervated by a branch of the tibial nerve

e. A patient with inability to dorsiflex and evert the foot most likely has a lesion in the deep peroneal nerve

29. A 42-year-old woman undergoes a very long and complicated vaginal hysterectomy. When she wakes up, she has numbness in the anterior and medial aspect of the thigh and medial leg. She also has knee extension weakness and her patellar reflex is absent. Hip flexion seems to be strong. An EMG 3 weeks later demonstrates reduced saphenous SNAPs and fibrillations in the quadriceps femoris. Other muscles are spared, including the iliacus, thigh adductors, and the muscles distal to the knee. Which of the following is correct?

a. This is consistent with a femoral nerve injury distal to the inguinal ligament

b. This is consistent with a femoral nerve injury in the intrapelvic region

c. This patient has a lumbar plexopathy

d. This patient has a L2 and L3 radiculopathy

e. This patient has involvement of the nerve arising from the anterior divisions of L2, L3, and L4 spinal roots

30. A 32-year-old man presents to the clinic complaining of sensory loss in his feet. He had noticed he could not sense the water temperature with his toes when he stepped into the tub, and he had sustained abrasions to his feet several times and had not realized it until he saw his feet bleeding. On examination, there is loss of sensation to pinprick and temperature on the distal lower extremities bilaterally. Examination also reveals an orange color to his tonsils. Which of the following is incorrect regarding the diagnosis of this patient?

a. Patients with this disorder have low serum high-density lipoprotein and low-density lipoprotein with elevated triglycerides

b. It can involve the peripheral nervous system as well as the reticuloendothelial system

c. It was first described in the Tangier islands off the coast of Virginia

d. It is autosomal dominant in inheritance

e. It results from a mutation in an adenosine triphosphate transporter protein

31. A 62-year-old man presents with sensory ataxia, painful paresthesias, and sensory deficits in his hands and feet. He is found to have a lung mass, and biopsy shows a small cell neoplasm. Which of the following antibodies will most likely be positive?

a. Anti-voltage-gated calcium channel

b. Anti-Yo

c. Anti-Ri

d. Anti-MAG

e. Anti-Hu

32. A 70-year-old woman presents 3 months after a right hip dislocation and fracture for evaluation of persistent right foot drop. EMG/NCS demonstrates reduced sural and superficial peroneal SNAP and reduced CMAPs in peroneal nerve–innervated muscles. There are fibrillations and reduced motor unit potential recruitment in the tibialis anterior, extensor hallucis, extensor digitorum brevis, peroneus longus, and peroneus brevis. Some fibrillations are noticed in the tibialis posterior and flexor digitorum longus, as well as the short head of the biceps femoris muscles. Where is the lesion?

a. Common peroneal nerve

b. Deep peroneal nerve

c. L5 nerve root

d. Sciatic nerve

e. Tibial nerve

33. Which of the following treatment options is not used for treatment of chronic inflammatory demyelinating polyneuropathy?

a. Prednisone daily

b. Intravenous immunoglobulin

c. Plasmapheresis

d. Natalizumab

e. Azathioprine

34. Which of the following is correct regarding the tibial nerve?

a. A lesion at the level of the tarsal tunnel manifests with plantarflexion weakness

b. The calcaneal branch innervates the abductor hallucis and flexor digitorum brevis

c. The sural nerve arises from the tibial nerve after it passes through the tarsal tunnel

d. The tibialis anterior is an L5 and tibial nerve–innervated muscle

e. It innervates the semimembranosus, semitendinosus, and long head of the biceps femoris

Questions 35–36

35. A 52-year-old man presents with weakness without any sensory symptoms. He initially developed left wrist drop about a month ago. Three weeks ago, he started complaining of right-hand weakness, and last week, he developed right-foot drop. Reflexes are absent in the upper limbs and right ankle. His right patellar reflex is diminished, but left lower extremity reflexes are normal. Anti-GM1 antibodies are present. His CSF shows a protein level of 40 mg/dL (normal up to 45 mg/dL) and cell count of 3/mm3 (normal up to 5 lymphocytes/mm3). There is evidence of motor conduction block in many peripheral nerves. Which of the following is the most likely diagnosis?

a. Chronic inflammatory demyelinating polyneuropathy

b. Acute inflammatory demyelinating polyeneuropathy

c. Multifocal motor neuropathy

d. Multifocal acquired demyelinating sensory and motor neuropathy

e. Subacute inflammatory demyelinating polyneuropathy

36. Which of the following is correct regarding this condition?

a. Sensory NCS are usually abnormal

b. Anti-GM1 antibodies should be positive to make the diagnosis

c. Anti-GM1 antibody titers predict response to treatment

d. The presence of conduction block does not correlate with response to treatment

e. This condition is very responsive to steroid treatment

37. Regarding the ulnar nerve, which of the following is incorrect?

a. It does not innervate any upper arm muscles

b. It is a continuation of the lateral cord

c. It is most susceptible to injury at the medial epicondyle

d. It passes through Guyon’s canal to reach the hand

e. It predominantly carries C8 and T1 fibers

38. A 49-year-old man presents with 2 months of gradually progressive symmetric proximal and distal weakness, paresthesias, and areflexia. A chronic inflammatory demyelinating polyneuropathy is considered as the possible diagnosis. CSF shows WBC of 25/mm3 (normal up to 5 lymphocytes/mm3) and a protein level of 150 mg/dL (normal up to 45 mg/dL). Which of the following tests does not need to be ordered?

a. HIV testing

b. Monoclonal protein analysis

c. Hepatitis C antibody

d. Acetylcholine receptor antibodies

e. EMG/NCS

39. Which of the following inherited neuropathies are not autosomal dominant?

a. Charcot-Marie-Tooth type (CMT) 1

b. Hereditary sensory and autonomic neuropathy type 1

c. CMT4

d. Familial amyloid polyneuropathies

e. Hereditary neuropathy with liability to pressure palsy

40. A 40-year-old man presents with pain in the right lower extremity radiating from the buttock down to his foot. He has pain and sensory deficit along the posterior thigh, leg, and lateral aspect of the foot. There is weakness on plantarflexion. Patellar reflex is normal, but the ankle reflex is depressed on the right side. Which of the following is the most likely diagnosis?

a. L2 and L3 radiculopathy

b. L4 radiculopathy

c. L5 radiculopathy

d. S1 radiculopathy

e. Peroneal neuropathy

41. A 40-year-old man presents to the clinic complaining of left-hand weakness. He complains of significant loss of fine motor coordination in the left hand. On examination, he has atrophy of the intrinsic hand muscles, weakness of wrist flexion in an ulnar direction, flexion at the distal interphalangeal joint of fourth and fifth digits, and abduction and adduction of all the fingers. There is loss of sensation over the hypothenar eminence and the fourth and fifth digits, but not more proximally. Proximal arm muscle strength, forearm flexion and pronation, and flexion of the second and third digits at both the proximal and distal interphalangeal joints are of normal strength. Thumb abduction is mildly weak. On attempt to make a fist, there is hyperextension at the metacarpophalangeal joint of the fourth and fifth digits and flexion at the proximal but not distal interphalangeal joints. Ulnar nerve CMAPs are shown in Figure 9.4. What is the most likely diagnosis in this patient?

FIGURE 9.4 Ulnar nerve motor NCS (Courtesy of Dr. Robert Shields)

a. A C8 radiculopathy

b. Ulnar neuropathy at the wrist

c. Medial cord lesion

d. A C7 radiculopathy

e. Ulnar neuropathy at or above the elbow

42. A 65-year-old man with type 2 diabetes mellitus presents with right lower extremity pain and weakness. He reports that symptoms began with severe low back pain radiating down his right hip and thigh. He then noticed difficulty flexing his hip and extending his knee. On examination, he has atrophy of the right thigh muscles and severe weakness of the hip flexors, adductors, quadriceps, and hamstring muscles on the right side. There are sensory deficits in the anteromedial thigh. His patellar reflex is absent on the same side. Which of the following is the most likely diagnosis?

a. Small-fiber diabetic neuropathy

b. Large-fiber diabetic neuropathy

c. Diabetic autonomic neuropathy

d. Diabetic mononeuropathy

e. Diabetic amyotrophy

43. Which of the following is incorrect regarding the radial nerve?

a. It is a continuation of the posterior cord

b. It carries C5, C6, C7, and C8 fibers

c. It innervates all three heads of the triceps muscle

d. It provides sensory innervation to most of the posterolateral arm and forearm

e. All of the forearm muscles innervated by the radial nerve are forearm extensors

44. A 50-year-old woman is admitted for asthma exacerbation, and suffers multiple medical complications during her hospitalization. She is bedridden for more than 4 weeks and is found to have a right foot drop. On neurologic evaluation, there is weakness in dorsiflexion and eversion of the foot. An NCS is performed and demonstrates reduced CMAP amplitude of the tibialis anterior and extensor digitorum brevis, as well as reduced SNAP amplitude of the superficial peroneal. EMG shows fibrillations in the tibialis anterior, extensor hallucis, extensor digitorum brevis, and peroneus longus. Short head of the biceps femoris is normal. Other muscles tested demonstrated no abnormalities. Which of the following is the most likely diagnosis?

a. Common peroneal nerve

b. Deep peroneal nerve

c. L5 nerve root

d. Sciatic nerve

e. Tibial nerve

45. A 52-year-old truck driver since his teenage years presents with tingling in the fourth and fifth digits of his left hand. The tingling is mild but annoying to him. On examination, there is reduced sensation to all modalities on the dorsal and palmar aspect of the fourth and fifth digits from the wrist crease to the finger tips, with preserved strength in all muscle groups. Which of the following statements is correct?

a. This man has carpal tunnel syndrome

b. This man has an ulnar neuropathy at the elbow

c. This man should be referred to a surgeon

d. EMG is expected to show fibrillation potentials in the C6 and C7 myotomes

e. Conservative management frequently fails in this type of disorder

46. Which of the following complications is associated with diabetic neuropathy?

a. Foot ulcers

b. Arthropathy affecting the ankles

c. Sensory ataxia

d. Acute third nerve palsy

e. All of the above

47. A 30-year-old obese man who works as a mechanic comes for evaluation of pain and numbness in the lateral aspect of his thigh. There are no motor deficits. Which of the following is the most likely structure involved?

a. Lateral femoral cutaneous nerve

b. Femoral nerve

c. Saphenous nerve

d. Obturator nerve

e. Lumbosacral plexus

48. A 50-year-old man undergoes coronary artery stenting performed through a femoral artery puncture. After the procedure, he becomes hypotensive and requires admission to the ICU. His hematocrit drops and he needs blood transfusion. He later notices pain with hip flexion and numbness in the anterior and medial thigh. He also has difficulty flexing the hip and extending the knee, and his patellar reflex is absent. Three weeks later, NCS show a reduced saphenous SNAP. On needle EMG, fibrillation potentials are seen in the iliacus and quadriceps muscles. Thigh adductors and muscles below the knee show no abnormalities. Which of the following is correct?

a. This is consistent with a femoral nerve injury at the inguinal region

b. This is consistent with a femoral nerve injury in the intrapelvic region

c. This patient has a lumbar plexopathy

d. This patient has an obturator nerve injury

e. This patient has involvement of the nerve arising from the anterior divisions of L2, L3, and L4 spinal roots

49. A 50-year-old woman with a 10-year history of type 2 diabetes mellitus presents with burning pain in both feet. Sensation to vibration and proprioception is preserved, as are the deep tendon reflexes. Which of the following most likely explains her symptoms?

a. Small fiber diabetic neuropathy

b. Large fiber diabetic neuropathy

c. Diabetic polyradiculoneuropathy

d. Diabetic mononeuropathy

e. Diabetic amyotrophy

50. A 73-year-old man with poorly controlled diabetes presents with complaints of painless weakness of extension of the fingers in the left hand. On examination, forearm extension and wrist extension and abduction (wrist extension in a radial direction) are normal in strength, but wrist extension and adduction (wrist extension in an ulnar direction) are weak. Forearm supination is weak, particularly when tested with the forearm extended, but there is no pain with active supination. Finger extension at the metacarpophalangeal joints is also weak, as is thumb abduction in the plane of the palm, and thumb extension at the interphalangeal and metacarpophalangeal joint. Sensory examination is normal. On NCS, the superficial sensory radial nerve is normal. Triceps deep tendon reflex is normal. What is the most likely diagnosis?

a. Radial neuropathy at the spiral groove

b. C7 radiculopathy

c. Posterior interosseus nerve palsy

d. Radial neuropathy at the elbow

e. Supinator syndrome

51. A 38-year-old man presents with pain in the left lower extremity. The pain and sensory deficits are localized to the anterior thigh and medial leg. There is weakness on hip flexion, knee extension, and ankle dorsiflexion. The patellar reflex on the left is depressed and the ankle reflex is normal. Saphenous SNAP is normal. On EMG, fibrillation potentials are seen in the iliacus, vastus lateralis and medialis, rectus femoris and tibialis anterior muscles. Paraspinal fibrillations are also appreciated. Which of the following is the most likely diagnosis?

a. Lumbar plexopathy

b. L2, L3 and L4 radiculopathy

c. L5 radiculopathy

d. S1 radiculopathy

e. Femoral neuropathy

52. A 24-year-old man undergoes minimally invasive mitral valve repair. He does well with normal arm strength until 3 days postoperatively when he begins experiencing severe shoulder and arm pain. The pain resolves 7 days later, but his arm becomes weak over ensuing days. He has weakness of shoulder abductors, arm external rotators, forearm flexors, forearm pronators, and finger flexors. What is the most likely diagnosis in this patient?

a. Cervical-brachial-pharyngeal variant of Guillian-Barre syndrome

b. Acute brachial plexitis (Parsonage-Turner syndrome)

c. Post-sternotomy brachial plexus lesion

d. Infectious polyradiculitis

e. An axillary neuropathy due to positioning during the operation

Questions 53–54

53. A 32-year-old man sustains a knife stab wound to the left antecubital fossa. He is brought to the emergency department where he is treated for brachial artery hemorrhage and is otherwise stabilized. On follow-up 8 weeks later, he denies any pain in his limb. He has Medical Research Council grade 2/5 strength in forearm pronation, thumb opposition, flexion, and abduction, flexion at the distal interphalangeal joint of the second digit, and flexion at the proximal interphalangeal joint of the second to fifth digits. Wrist flexion is 4/5, but the hand deviates in an ulnar direction during flexion. Flexion at the distal interphalangeal joint of the fourth and fifth digits is normal. Sensation is markedly reduced on the distal dorsal aspect of the first three digits and on the lateral (radial) aspect of the palm and first three digits, as well as the lateral (radial) aspect of the fourth digit. When asked to make a fist, the patient can barely flex the thumb, can partially flex the second digit, and has normal flexion of the fourth and fifth digits. What is the most likely diagnosis in this patient?

a. Complete median nerve palsy at the level of the antecubital fossa

b. Ischemic monomelia

c. Anterior interosseus nerve syndrome

d. A medial cord lesion

e. A C7 radiculopathy

54. A 52-year-old woman presents to the neuromuscular clinic complaining of finger weakness in the right hand. She reports difficulty holding a teacup with the right hand using a pincer grasp. She denies any sensory symptoms. On examination of the right upper limb, there is weakness of flexion at the distal interphalangeal joint of the second and third digits, weakness of thumb flexion, and weakness of forearm pronation when the forearm is fully flexed. Otherwise, all other muscles groups are of normal strength, and there is no evidence of sensory loss. What is the most likely diagnosis in this patient?

a. Complete median nerve palsy at the level of the antecubital fossa

b. Ischemic monomelia

c. Anterior interosseus nerve syndrome

d. A medial cord lesion

e. A C7 radiculopathy

Questions 55–56

55. An lumbar puncture is obtained on a patient with suspected Guillain-Barre syndrome. Which of the following findings do you expect?

a. Increased lymphocytes with normal protein

b. Increased neutrophils with normal protein

c. Increased protein with normal cell count

d. Abnormal CSF production of immunoglobulins, with the presence of oligoclonal bands and myelin basic protein

e. Increased RBCs

56. Which of the following findings does not occur in Guillain-Barre syndrome?

a. MRI showing gadolinium enhancement in the cauda equina

b. Abnormal or prolonged F-responses on NCS

c. Persistently normal distal latencies on NCS

d. Conduction block

e. Abnormal H-reflex

57. A 62-year-old woman presents to the neuromuscular clinic with complaints of painless right upper extremity weakness and sensory loss. She has a history of cancer of the right breast for which she had undergone surgery, chemotherapy, and radiation 3 years earlier. On examination, she has weakness in arm abduction and adduction, forearm flexion and extension, wrist flexion and extension, and sensory loss over the entire arm. EMG confirms involvement of all muscles examined, and myokymic discharges are evident on the EMG. Her left arm and lower extremities are normal. What is the most likely diagnosis in this patient?

a. Carcinomatous invasion of the brachial plexus

b. Chemotherapy-induced neuropathy

c. Radiation-induced brachial plexopathy

d. Paraneoplastic sensorimotor neuropathy

e. Cervical spine stenosis

58. A 56-year-old man with a history of diarrhea 2 weeks prior, presents with 4 days of difficulty walking and diplopia. On examination, he is very unsteady, and cannot walk straight. The motor examination shows full strength; however, the ankle and patellar reflexes are absent. Which of the following antibodies may be involved?

a. GM1

b. GD1a

c. GD1b

d. GQ1b

e. GalNac-GD1a

59. A 19-year-old man is hired as a packager on an assembly line. His main job is to take heavy objects off the conveyer belt and place them in a box that immediately follows the object on the belt. He does this hundreds of times a day using his right arm. He presents to the company physician 3 months after he starts his job complaining of a deep aching pain in the proximal right forearm that worsens with forearm pronation against resistance. On examination, motor testing is limited due to pain, but weakness of wrist flexion, thumb abduction, and flexion of the second digit is apparent. Strength of forearm pronation appears normal. What is the most likely diagnosis?

a. Complete median nerve palsy at the elbow

b. Pronator teres syndrome

c. Anterior interosseus nerve syndrome

d. A medial cord lesion

e. A C7 radiculopathy

60. Which of the following is incorrect regarding the management of patients with carpal tunnel syndrome?

a. In patients without known risk factors for carpal tunnel, and in the appropriate clinical setting, testing for hypothyroidism and diabetes is indicated

b. Acromegaly can be a cause of carpal tunnel syndrome

c. Amyloidosis can be a cause of carpal tunnel syndrome

d. Surgical therapy is indicated in asymptomatic carpal tunnel syndrome

e. Surgical release for carpal tunnel syndrome is indicated in severe cases

Questions 61–62

61. A 34-year-old woman fractures her left tibia. Because of several complications, she requires the use of crutches over several months. She later presents to the neuromuscular clinic with complaints of weakness in her right arm. On examination, she has weakness in forearm extension, wrist extension, and finger extension, as well as sensory loss along the posterolateral arm, forearm, and dorsolateral hand. The triceps deep tendon reflex is absent. Arm adduction and abduction, forearm pronation, and wrist flexion are normal in strength. What is the most likely diagnosis in this patient?

a. Radial neuropathy at the spiral groove

b. Radial neuropathy at the axilla

c. Radial neuropathy at the elbow

d. C7 radiculopathy

e. Posterior cord lesion

62. A 37-year-old man is involved in a motor vehicle accident and fractures his right humerus. He later presents to a clinic with complaints of weakness in wrist and finger extension. On examination, forearm extension is strong. Wrist extension in both the radial and ulnar direction and finger extension are weak, and there is loss of sensation on the lower lateral arm and posterior forearm. What is the most likely diagnosis in this patient?

a. C7 radiculopathy

b. Radial neuropathy at the axilla

c. Radial neuropathy at the elbow

d. Radial neuropathy at the spiral groove

e. Posterior cord lesion

63. A 52-year-old man is diagnosed with a neck mass, and undergoes excision of the mass, with intraoperative frozen sections showing adenocarcinoma with positive margins. The surgeon elects to pursue a radical neck dissection with extensive excision. Following initial recovery, the patient reports weakness in raising his right arm above his head. On examination, his right shoulder is noted to be hanging at a lower level as compared to the left. Right arm abduction to 90 degrees is normal, but there is weakness of arm abduction above that level. Shoulder shrug is Medical Research Council 4/5 in strength. There is weakness in head turning to the left. Otherwise, upper extremity examination is normal, and there is no sensory loss in the head, neck, or arm region. Cranial nerve examination and gag reflex are otherwise normal as well. What is the most likely diagnosis in this patient?

a. Lesser occipital nerve injury

b. Jugular foramen syndrome

c. Cervical plexus injury

d. Spinal accessory nerve injury

e. Greater occipital nerve injury

64. Which of the following presentations of nervous system involvement can occur with diabetes?

a. Small-fiber neuropathy

b. Large-fiber neuropathy

c. Autonomic neuropathy

d. Only a and c

e. All of the above

65. A 48-year-old man presents with numbness, tingling, and weakness. He initially experienced weakness in the right hand and later developed weakness in the left hand and left foot. He also has painful paresthesias in the right hand and left foot. Deep tendon reflexes are normal in the right lower limb, but decreased everywhere else. His CSF shows a protein level of 75 mg/dL (normal up to 45 mg/dL) and cell count of 3/mm3 (normal up to 5 lymphocytes/mm3). There is evidence of conduction block in many peripheral nerves and abnormal sensory NCS. Which of the following is the most likely diagnosis?

a. Chronic inflammatory demyelinating polyneuropathy

b. Acute inflammatory demyelinating polyeneuropathy

c. Multifocal motor neuropathy

d. Multifocal acquired demyelinating sensory and motor neuropathy

e. Subacute inflammatory demyelinating polyneuropathy

66. A 19-year-old man is arrested during attempted robbery. While he is transported to the jail, he is agitated, and keeps pulling on the handcuffs. The prison physician is later called to see him for numbness of his right hand, because there was concern that he was having a stroke. On examination, he had decreased sensation over the dorsolateral aspect of the right hand, with normal motor strength and normal sensation in other areas. What is the most likely diagnosis?

a. Acute ischemic stroke in the hand area of the postcentral sulcus

b. Carpal tunnel syndrome

c. The symptoms are not consistent with any particular neurologic disorder; this man is malingering

d. Superficial sensory radial neuropathy

e. C8 radiculopathy

67. A 32-year-old man is involved in a motorcycle accident and sustains an anterior shoulder dislocation. Six weeks later, he presents to the clinic complaining of weakness in flexion of his right forearm at the elbow. On examination, right forearm flexion strength is Medical Research Council grade 2/5, and there is minimal palpable contraction of arm muscles during attempted forearm flexion. Forearm supination is 2/5 in strength with the forearm flexed, and 4/5 in strength with the forearm extended. The biceps deep tendon reflex is absent. Arm adduction is normal in strength. Forearm pronation, and distal motor strength, such as wrist flexion, is normal. There is diminished sensation over the lateral aspect of the forearm, but sensation over the thumb and second digit is normal. What is the most likely diagnosis in this patient?

a. Biceps tendon rupture

b. C6 radiculopathy

c. Median neuropathy at the elbow

d. Lateral cord lesion

e. Musculocutaneous neuropathy

68. A 62-year-old woman presents for evaluation of severe pain and paresthesias in her feet. Sensory examination shows impairments to pinprick and temperature below the ankles. Motor examination and reflexes are preserved. A small-fiber neuropathy is considered. Which of the following is incorrect regarding this condition?

a. Diabetes is a common cause of this condition

b. Quantitative sudomotor axon reflex test can be used for diagnosis

c. Skin biopsy and intraepidermal nerve fiber density can be used for diagnosis

d. Thermoregulatory sweat test can be used for diagnosis

e. EMG/NCS are usually abnormal

69. A 10-year-old girl falls off a ledge and fractures the proximal humerus. After her cast is removed, she is noted to have weakness of the arm, and neurologic consultation is requested. On examination, she is able to abduct the arm up to around 30 degrees, but not beyond that. Arm flexion is Medical Research Council grade 2/5. Sensory examination reveals a small area of hypoesthesia on the lateral aspect of the upper arm, but is otherwise normal. Arm adduction, forearm pronation, and forearm, wrist, and finger extension, are of normal strength. What is the most likely diagnosis in this patient?

a. Axillary neuropathy

b. Radial neuropathy

c. Posterior cord lesion

d. C5 radiculopathy

e. Middle trunk lesion

70. A 39-year-old man is admitted to the ICU with an acute pontine infarct. Work-up shows a dolichoectatic basilar artery with a partially thrombosed aneurysm. Echocardiogram reveals a dilated cardiomyopathy. On questioning of family members, it is revealed that since adolescence, he had complained of severe burning pains in his hands and feet with heat exposure or exercise, but his pediatrician had attributed them to growing pains and they had not been fully investigated. On examination, he has several dark-purplish punctuate lesions on his trunk and scrotum. His father suffered from burning hands and feet and in middle age required dialysis for renal failure of unclear etiology. Which of the following is correct regarding the disorder depicted in this case?

a. It is due to a deficiency of the enzyme arylsulfatase A

b. It is due to a deficiency in the enzyme α-galactosidase A

c. It is autosomal dominant in inheritance

d. The stroke in this patient is likely unrelated to his underlying condition

e. It is a glycogen storage disease

71. A 38-year-old athlete sustains an injury while playing football. Eight weeks later, he presents to a neuromuscular clinic with right arm weakness. He has Medical Research Council motor power of 4/5 in external rotation of the arm, forearm flexion, and shoulder abduction. There is sensory loss over the lateral aspect of the arm and forearm. The biceps deep tendon reflex is absent. On EMG, there are fibrillation potentials in the biceps, brachialis, deltoid, brachioradialis, supraspinatus, and rhomboid muscles. EMG examination of the triceps, pronator teres, brachioradialis, and intrinsic hand muscles is normal. Which of the following best explains this patient’s weakness?

a. An upper brachial plexus trunk lesion

b. An axillary nerve lesion

c. A C5 and C6 root lesion

d. A lesion to the musculocutaneous nerve

e. A lesion to the lateral cord of the brachial plexus

72. A 45-year-old athlete who spends several hours a day weight lifting presents with deep, aching shoulder pain and arm weakness. On examination, he has weakness of shoulder abduction and external rotation with the forearm flexed while the elbow is stabilized against the patient’s side. Sensory examination is normal, and arm adduction is normal. Winging of the scapula is not evident. Forearm flexion is normal, and the biceps deep tendon reflex is normal. This is most consistent with

a. Injury to the thoracodorsal nerve

b. Suprascapular nerve entrapment

c. Long thoracic nerve injury

d. C5 radiculopathy

e. An upper trunk lesion

73. A 55-year-old man with mild truncal obesity, hyperlipidemia, and long-standing diabetes comes referred from gastroenterology. Four months ago, he developed bulging in the right lower abdomen with a patch of numbness and a sensation of burning to touch in this area. You examine him and find that he has an absent superficial abdominal reflex in the right lower quadrant of the abdomen, as well as patchy reduction to pinprick and light touch on both sides of the abdominal wall at approximately T10 to T12. You feel his history is most consistent with the following:

a. Spinal cord ischemia

b. Stretch neuropathy due to obesity

c. Femoral nerve injury

d. Thoracoabdominal polyradiculopathy

e. Malingering

74. An 18-year-old woman presents to the clinic complaining of right-hand weakness. She reports that for many months while cheerleading, with her arms abducted and externally rotated and forearms flexed (holding up her pom-poms) she feels a dull ache and tingling in her hand, mainly in the fourth and fifth digits. More recently, the aching has become more continuous and she has noted some numbness of the medial forearm. Examination shows weakness of thumb abduction away from the plane of the palm, and weakness of finger abduction and adduction, with subtle atrophy of the hand muscles. There is sensory loss over the medial aspect of the hand and forearm. What is the most likely diagnosis in this patient?

a. Ulnar neuropathy at the elbow

b. Carpal tunnel syndrome

c. Neurogenic thoracic outlet syndrome

d. C8 radiculopathy

e. Median neuropathy at the elbow

Questions 75–77

75. A 20-year-old woman presents to the clinic with debilitating shooting pains in the feet and legs. Examination shows loss of sensation to pinprick and temperature, with mildly reduced sensation to vibration and fine touch. Proprioception is intact in the fingers and toes. She is fearful that she is developing the condition that her father and paternal grandfather suffered from; they both are deceased, and she does not recall details, but remembers that as far back as she could remember, they had problems with foot ulcerations and odd deformities in their toes and ankles, and her father had required a below-the-knee amputation. On directed questioning, she admits to heat intolerance and feels she does not sweat the way other people normally would. What is the most likely diagnosis in this patient?

a. Hereditary sensory and autonomic neuropathy (HSAN) type 1

b. Charcot-Marie-Tooth 1A

c. HSAN type 2

d. HSAN type 3

e. HSAN type 4

76. The parents of a 1-year-old boy bring him to the clinic with multiple concerns. In the first month of life he was noted to have trouble swallowing and suffered from aspiration pneumonia, and a percutaneous gastrostomy tube was placed. During that admission, his blood pressure was noted to be “all over the place.” His parents noticed that when he cried, he would not produce tears, but would become very flushed and sweat dramatically. What is the most likely diagnosis in this patient?

a. Hereditary sensory and autonomic neuropathy (HSAN) type 1

b. Charcot-Marie-Tooth 1A

c. HSAN type 2

d. HSAN type 3

e. HSAN type 4

77. A 2-year-old girl is brought to the pediatric neurologist by her parents. She had suffered from recurrent fevers since childhood, but no matter what fluctuations occurred in her body temperature, she would never sweat. An infectious cause to her fevers could not be identified. They noticed that she did not cry when she received her vaccinations, and since she had started walking, if she bumped into an object or fell, she did not seem to feel pain, even when she sustained a significant abrasion or bruise. What is the most likely diagnosis in this patient?

a. Hereditary sensory and autonomic neuropathy (HSAN) type 1

b. Charcot-Marie-Tooth 1A

c. HSAN type 2

d. HSAN type 3

e. HSAN type 4

78. Which of the following conditions does not present as mononeuritis multiplex?

a. Vasculitic neuropathy

b. Diabetes

c. Guillain-Barre syndrome

d. Cryoglobulinemic vasculitis

e. Neuropathy associated with HIV

79. A 37-year-old man presents with right wrist drop and left foot drop, associated with painful paresthesias, rash, and arthralgias. A blood sample is sent for evaluation, detecting precipitation of protein when exposed to cold temperature. Which of the following is incorrect?

a. Commonly associated with hepatitis C

b. This patient has multiple mononeuropathies associated with cryoglobulinemia

c. Complement levels are elevated

d. May be associated with HIV

e. Multiple myeloma may be associated

80. A 22-year-old man presents to the clinic with complaints of severe pain and redness of the limbs with exercise or whenever the weather is warm. When these episodes occur, he goes home and sits in a tub of ice water until he cools down. These episodes have become so severe that he minimizes physical exertion despite having a love for sports, and is actually thinking of moving north to prevent heat exposure. He is otherwise healthy, and physical examination is normal. His father suffers from similar symptoms. Routine laboratory testing, including complete blood count and fasting glucose, are normal. What is the most likely diagnosis in this patient?

a. Hereditary sensory and autonomic neuropathy type 1

b. Primary erythromelalgia

c. Secondary erythromelalgia

d. Fabry’s disease

e. Small fiber neuropathy associated with glucose intolerance

81. A 57-year-old man presents with 3 months of gradually progressive neurologic symptoms that began with numbness and dysesthesias in his hands and feet, which have worsened. More recently, he has been having difficulty walking, especially when it is dark, and reports that he feels unsteady when he closes his eyes. On examination, he has ataxia in all four limbs when he closes his eyes, patchy sensory deficits in various parts of his body without a distal-to-proximal gradient, and areflexia. His strength is normal. NCS demonstrate bilateral asymmetric reduction of SNAPs in various sensory nerves, more in the upper than lower extremities, with normal CMAPs. Which of the following is incorrect regarding this condition?

a. SSA and SSB antibodies should be obtained

b. Anti-Hu antibodies should be obtained

c. Lung cancer should be ruled out

d. The dorsal root ganglia are likely involved

e. Pyridoxine should be supplemented at high doses

82. A 36-year-old man is standing on the ledge of the balcony of his fourth-floor apartment and is putting up Christmas lights. He starts to fall, but luckily grabs onto the railing of the balcony with his left hand, and is left barely hanging from the railing with his finger tips. His brother hears him crying for help and assists him. He experiences severe pain in his arm, but is relieved to have not fallen to the ground. Over the next several days, he notices difficulty with fine motor movements of his left hand. On examination, he has weakness of finger flexion at the proximal and distal interphalangeal joint of the second to fifth digits, weakness of finger abduction and adduction, weakness of thumb abduction away from the plane of the palm, and weakness of wrist flexion in an ulnar (medial) direction. There is sensory loss over the medial forearm and fourth and fifth digits, but sensation is otherwise intact. Arm and forearm flexors and extensors as well as wrist and finger extensors are strong. What is the most likely diagnosis in this patient?

a. Ulnar neuropathy at the wrist

b. Median neuropathy at the elbow

c. Radial neuropathy

d. Lower trunk lesion

e. Lateral cord lesion

83. A 14-year-old girl presents to the emergency department with severe abdominal pain, nausea, and vomiting. Routine laboratory testing including liver enzymes is normal. She is admitted to the hospital and treated with intravenous fluids. Two days later, she is witnessed to have a generalized tonic-clonic seizure. She subsequently begins experiencing hallucinations and delusions. MRI of the brain with contrast and CSF analysis are normal. Four days after admission, she is noted to have mild weakness of wrist extension; one week later, examination shows bilateral wrist drop. What is the most likely diagnosis in this patient?

a. Wilson’s disease

b. Systemic lupus erythematosuis

c. Acute inflammatory demyelinating polyneuropathy

d. Acute disseminated encephalomyelitis

e. Acute intermittent porphyria

84. A 32-year-old man is being evaluated by a company physician prior to employment. The physician notices the patient’s arm is weak; on further questioning, the patient states, “I’ve been like this since I was born, my mom said that’s just how I was made.” On examination, he holds his right arm close to his body, internally rotated, with his wrist and fingers flexed. Arm abduction and flexion are weak. Arm extension is normal in strength. Biceps deep tendon reflex is absent, but triceps deep tendon reflex is normal. There is sensory loss along the lateral half of the arm and forearm and the thumb and index finger. EMG shows no evidence of denervation in the rhomboids. What is the most likely diagnosis in this patient?

a. An axillary neuropathy

b. An upper trunk lesion

c. A C7 root lesion

d. A middle trunk lesion

e. A C5 and C6 root lesion

85. A 39-year-old woman with type 1 diabetes mellitus presents for evaluation. She has numbness in a glove and stocking distribution affecting hands and feet. She has no pain. There is loss of sensation to vibration and proprioception, and she is hyporeflexic distally. Which of the following most likely explains her symptoms?

a. Small fiber diabetic neuropathy

b. Large fiber diabetic neuropathy

c. Diabetic polyradiculoneuropathy

d. Diabetic mononeuropathy

e. Diabetic amyotrophy

86. A 60-year-old woman presents with 2 months of pain radiating from her buttock down to her right leg. She has foot drop on the right, with severe foot dorsiflexion weakness. There is sensory deficit in the right lateral leg and dorsum of the foot. NCS show reduced peroneal CMAPs recording from the tibialis anterior and extensor digitor brevis muscles and normal superficial peroneal SNAP. Needle EMG shows fibrillations and reduced recruitment in the tibialis anterior, extensor digitorum brevis, extensor hallucis, peroneous longus, tibialis posterior, and flexor digitorum longus. Which of the following is the most likely diagnosis?

a. L5 radiculopathy

b. S1 radiculopathy

c. Common peroneal neuropathy

d. Deep peroneal injury

e. Sciatic nerve injury

Answer Key

1. d

2. e

3. c

4. e

5. d

6. b

7. d

8. c

9. e

10. b

11. b

12. a

13. d

14. c

15. a

16. b

17. d

18. a

19. b

20. d

21. a

22. d

23. e

24. e

25. c

26. d

27. d

28. a

29. a

30. d

31. e

32. d

33. d

34. e

35. c

36. d

37. b

38. d

39. c

40. d

41. e

42. e

43. e

44. a

45. b

46. e

47. a

48. b

49. a

50. c

51. b

52. b

53. a

54. c

55. c

56. c

57. c

58. d

59. b

60. d

61. b

62. d

63. d

64. e

65. d

66. d

67. e

68. e

69. a

70. b

71. c

72. b

73. d

74. c

75. a

76. d

77. e

78. c

79. c

80. b

81. e

82. d

83. e

84. b

85. b

86. a

Answers

1. d

The history and examination of the patient in question 1 are consistent with giant axonal neuropathy (GAN). This is a rare autosomal recessive disorder that manifests in early childhood. It affects intermediate filaments of both the central and peripheral nervous system, leading to a sensorimotor neuropathy, corticospinal tract involvement with upper motor neuron signs, and optic atrophy leading to vision loss. The characteristic gait includes walking on the inner edges of the feet. The integument is also involved, and patients often have tightly curled hair. The neuropathy is predominantly axonal, and neuropathologic analysis of nerves from patients with this disorder reveal pathognomonic findings of large focal axonal swelling that contain tightly packed disorganized neurofilaments. This disorder is due to mutations in the GAN gene that encodes for gigaxonin, which is involved in cross-linking of intermediate filaments. It is progressive, and death typically occurs by adolescence.

The clinical history, particularly the hair findings, characteristic gait, evidence of CNS involvement, and neuropathologic findings distinguish GAN from Dejerine-Sottas syndrome, or Charcot-Marie-Tooth type (CMT) type 3 (discussed in questions 21 and 22), CMT4 (discussed in question 39), and CMT2A (discussed in questions 21 and 22), which also manifest in childhood and can be quite severe. Metachromatic leukodystrophy (MCL) (discussed in Chapter 14) is also a severe neurodegenerative disorder that manifests in early life; clinical manifestations include both central and peripheral nervous system manifestations. MCL is distinguished from GAN on the basis of the neuropathologic and imaging features, as well as biochemical studies, including elevated urine sulfatides in MCL (see Chapter 14).

Bradley WG, Daroff RB, Fenichel GM, et al. Neurology in Clinical Practice, 5th ed. Philadelphia, PA: Elsevier; 2008.

2. e

This patient’s history, examination, and laboratory testing are consistent with Refsum’s disease (RD). RD is an autosomal dominant peroxisomal disorder that results from a defect in an enzyme involved in fatty acid metabolism, leading to accumulation of an intermediate in this pathway, phytanic acid. Clinical manifestations include retinitis pigmentosa (with night blindness and visual field constriction), cardiomyopathy, and skin changes. Neurologic manifestations include neuropathy, hearing loss, anosmia, ataxia, and cerebellar signs. The neuropathy is a large-fiber sensorimotor neuropathy. The presence of overriding toes due to a shortened fourth metatarsal may aid in the diagnosis. Treatment includes dietary modification to reduce dietary intake of phytanic acid.

Clinical features of the other disorders listed also include retinitis pigmentosa and neuropathy, but elevated phytanic acid levels are diagnostic of Refsum’s disease. In myoneurogastrointestinal encephalopathy (MNGIE), intestinal pseudo-obstruction is a prominent feature; other features include ophthalmoparesis and a demyelinating neuropathy. MNGIE is due to a mutation in the thymidine phosphorylase gene.

Abetalipoproteinemia, also known as Bassen-Kornzweig syndrome, is an autosomal recessive disorder that results in defective triglyceride transport, leading to abnormal very low density lipoprotein secretion. Fat malabsorption results in deficiencies in vitamins A, E, D, and K. Low levels of serum β-lipoprotein and vitamin E in the serum suggest the diagnosis. Peripheral smear shows acanthocytes. Clinical manifestations include retinitis pigmentosa, neuropathy, and ataxia.

Neurogenic muscle weakness, ataxia, and retinitis pigmentosa syndrome is a mitochondrial cytopathy that results from a mutation in the adenosine triphosphate 6 gene. Patients may present with a predominantly sensory axonal neuropathy. Other mitochondrial cytopathies including mitochondrial encephalopathy with lactic acidosis and stroke-like symptoms and disorders due to polymerase γ mutations can also lead to neuropathy.

Kearns-Sayre syndrome (KSS) is a mitochondrial disorder resulting from a mutation in mitochondrial DNA. It manifests before the age of 20 with retinitis pigmentosa, progressive opthalmoplegia, cardiac conduction defects, ataxia, myopathy, and hearing loss; neuropathy is not a prominent feature of KSS.

Bradley WG, Daroff RB, Fenichel GM, et al. Neurology in Clinical Practice, 5th ed. Philadelphia, PA: Elsevier; 2008.

Ropper AH, Samuels MA. Adams and Victor’s Principles of Neurology, 9th ed. New York: McGraw-Hill; 2009.

3. c, 4. e

Significant axon loss lesions produce reductions in action potential amplitudes and tend to have preserved or mildly reduced conduction velocities. The F-wave is obtained after supramaximal stimulation of a motor nerve.

NCS are classified into sensory and motor conduction studies. Sensory NCS are obtained by stimulating a sensory nerve while recording the transmitted potential at a different site along the same nerve. Three main measures can be obtained: SNAP amplitude, sensory distal latency, and conduction velocity. The SNAP amplitude (in microvolts) represents a measure of the number of axons conducting between the stimulation site and the recording site. Sensory distal latency (in ms) is the time that it takes for the action potential to travel between the stimulation site and the recording site of the nerve. The conduction velocity is measured in meters per second and is obtained dividing the distance between two sites of stimulation by the differences of the latencies between these two sites: Conduction velocity = Distance/(Proximal latency – Distal latency).

Motor NCS are obtained by stimulating a motor nerve and recording at the belly of a muscle innervated by that nerve. The CMAP is the resulting response, and depends on the motor axons transmitting the action potential, status of the neuromuscular junction, and muscle fibers. The CMAP amplitudes, motor latencies, and conduction velocities are routinely assessed and analyzed.

In general, for sensory and motor responses, a decrease in the amplitudes correlates with axon loss lesions. On the other hand, prolonged latencies and slow conduction velocities correlate with demyelination. Low amplitudes can result from demyelinating conduction block when the nerve stimulation is proximal to the block.

The F-wave and the H-reflex are late responses. The F-wave is obtained after supramaximal stimulation of a motor nerve while recording from a muscle. The electrical impulse travels antidromically (conduction along the axon opposite to the normal direction of impulses) along the motor axons toward the motor neuron, backfiring and traveling orthodromically (conduction along the motor axon in the normal direction) down the nerve to be recorded at the muscle. The H-reflex is the electrophysiologic equivalent of the ankle reflex (S1 reflex arc) and is obtained by stimulating the tibial nerve at the popliteal fossa while recording at the soleus. The electrical impulse travels orthodromically through a sensory afferent, enters the spinal cord, and synapses with the anterior horn cell, traveling down the motor nerve to be recorded at the muscle.

Katirji B. Electromyography in Clinical Practice: A Case Study Approach, 1st ed. St. Louis: Mosby; 1998.

Preston DC, Schapiro BE. Electromyography and Neuromuscular Disorders, 2nd ed. Philadelphia, PA: Elsevier; 2005.

5. d

Large polyphasic motor unit potentials (MUPs) are not seen in acute neuropathic lesions, but rather in chronic ones.

When performing needle EMG examination, insertional and spontaneous activity as well as voluntary MUP activity should be characterized. Insertional activity is recorded as the needle is inserted into a relaxed muscle. It is increased in denervated muscles and myotonic disorders, and is decreased when the muscle is replaced by fat or connective tissue and during episodes of periodic paralysis. Spontaneous activity is assessed with the muscle at rest, and examples include fibrillation potentials, fasciculation potentials, and myokymia and myotonic potentials. All spontaneous activity is abnormal.

MUPs are obtained while the needle is inserted into the muscle during voluntary contraction. Various characteristics are of consideration, including recruitment pattern and MUP parameters, such as duration, amplitude, and configuration. Recruitment is a measure of the number of MUPs firing during increased force of voluntary muscle contraction. In axon loss lesions, reduced recruitment is characterized by a less-than-expected number of MUPs firing rapidly. Early or rapid recruitment occurs in myopathic processes with loss of muscle fibers, in which an excessive number of short-duration and small-amplitude MUPs fire during the muscle contraction. With poor voluntary effort or with CNS disorders causing weakness, recruitment is reduced with normal MUPs firing at slow or moderate rates, sometimes in a variable fashion. In neuropathic disorders with denervation and reinnervation, MUPs disclose increased duration and amplitude, and may be polyphasic. In myopathic disorders, MUPs are of reduced duration and amplitude, and may also be polyphasic.

Katirji B. Electromyography in Clinical Practice: A Case Study Approach, 1st ed. St. Louis: Mosby; 1998.

Preston DC, Schapiro BE. Electromyography and Neuromuscular Disorders, 2nd ed. Philadelphia, PA: Elsevier; 2005.

6. b

In a radiculopathy, there are normal SNAPs despite sensory symptoms.

The dorsal root ganglion is located just outside the spinal canal within the intervertebral foramen. It has sensory unipolar neurons with preganglionic fibers that extend proximally and enter the spinal cord through the dorsal horns, projecting rostrally in the spinal cord. The postganglionic fibers project distally through the spinal nerves and peripheral nerves, carrying information from a dermatome. On the other hand, the motor fibers originate from the anterior horn cells within the spinal cord, projecting distally through spinal nerves and peripheral nerves, carrying motor innervation to a myotome.

A radiculopathy occurs from an intraspinal canal lesion resulting in damage of the preganglionic fibers, leaving unaffected the cell body in the dorsal root ganglia and the postganglionic fibers, and therefore, even though sensory symptoms are prominent, the SNAPs are normal.

An axon loss radiculopathy will also injure motor fibers in the intraspinal canal region, affecting the respective myotome. This leads to denervation, with fibrillation potentials seen 3 weeks after the onset of motor axon loss, decreased recruitment, and 3 to 6 months later, large and polyphasic motor unit potentials (MUPs). The presence of these large and polyphasic MUPs is dependent on reinnervation and collateral innervation, typically occurring in a proximal-to-distal fashion, with proximal muscles more successfully reinnervated as compared to distal muscles.

The H-reflex is the electrophysiologic equivalent of the ankle reflex, which is an S1 reflex, and this test is helpful in the evaluation of S1 radiculopathies.

Katirji B. Electromyography in Clinical Practice: A Case Study Approach. 1st ed. St. Louis: Mosby; 1998.

Preston DC, Schapiro BE. Electromyography and Neuromuscular Disorders, 2nd ed. Philadelphia, PA: Elsevier; 2005.

7. d, 8. c

The 2-Hz repetitive nerve stimulation shown in Figure 9.1 demonstrates a more than 10% decre- mental response of the CMAP amplitude, which is consistent with a diagnosis of myasthenia gravis (MG).

MG is a disorder of the neuromuscular junction, due to an antibody-mediated destruction of postsynaptic nicotinic acetylcholine receptors (discussed further in Chapter 10). Electrodiagnostic studies are important to diagnose this condition and to help differentiate it from other neuromuscular junction disorders. Sensory NCS are normal in MG, and motor NCS are usually normal as well. Whenever CMAPs are found to be low in amplitude, a presynaptic disorder such as Lambert-Eaton syndrome or botulism should be suspected. An increment in the CMAP amplitudes after exercise or rapid repetitive stimulation is a feature of a presynaptic disorder, and not of MG.

Repetitive stimulation is a helpful test in the diagnosis of MG, in which there is a decrement of CMAP amplitudes with slow repetitive nerve stimulation (2 to 3 Hz), with a decrement of greater than 10% being consistent with MG. The decremental responses occur due to a normal reduction in the release of acetylcholine after subsequent stimulation and the reduced availability of receptors from the disease, leading to a loss of end-plate potentials and reduction of the motor action potentials. In presynaptic neuromuscular junction disorders, rapid stimulation with frequencies of 20 to 50 Hz produces an incremental response by overcoming the efflux of calcium (which occurs within 100 to 200 ms); this is not seen in postsynaptic disorders where the limiting factor is at the postsynaptic membrane.

Jitter analysis by single-fiber EMG (SFEMG) is performed by recording with a single-fiber needle electrode positioned to detect potentials from two muscle fibers of the same motor unit. The variability of the interpotential interval between these two potentials is the jitter, and it is abnormal in MG due to delayed neuromuscular transmission. Neuromuscular blocking can also be detected, and is measured by the percentage of discharges in which one of the potentials is missing. SFEMG is highly sensitive but not specific for MG, being frequently abnormal in other neuromuscular junction disorders.

Moment-to-moment variation of the motor unit potentials (MUPs) may be present in MG, meaning that the MUPs vary in amplitude and configuration with successive discharges, due to blocking at some of the neuromuscular junctions of the muscle fibers composing the MUP.

Given the abnormal slow repetitive stimulation, as well as the normal NCS and needle examination on this patient, the diagnoses of myopathy and neuropathy are unlikely. The time frame of the presentation is not consistent with botulism.

Katirji B. Electromyography in Clinical Practice: A Case Study Approach, 1st ed. St. Louis: Mosby; 1998.

Preston DC, Schapiro BE. Electromyography and Neuromuscular Disorders, 2nd ed. Philadelphia, PA: Elsevier; 2005.

9. e

Jitter on single-fiber EMG is an indication of a neuromuscular junction abnormality, but it is not specific and cannot distinguish between myasthenia gravis and Lambert-Eaton myasthenic syndrome (LEMS).

LEMS is a disorder of neuromuscular transmission characterized by a reduced release of acetylcholine, and antibodies to presynaptic voltage-gated calcium channels. LEMS is frequently detected as a paraneoplastic syndrome associated with small cell lung carcinoma (discussed in Chapter 10). Electrodiagnostic studies are helpful in diagnosing this condition. Sensory NCS are normal, but CMAP amplitudes are usually low to borderline low at rest because many fibers fail to reach threshold after a stimulus, given inadequate release of acetylcholine vesicles. Brief exercise facilitates a release of acetylcholine and results in an increment in the CMAP amplitudes.

Patients with LEMS have a decremental response with slow repetitive stimulation, given that there is a decline in acetylcholine release with each stimulus, leading to loss of end-plate potentials and reduction of the motor action potential. However, these patients have an incremental response with rapid repetitive stimulation, given that the calcium availability in the presynaptic terminal is enhanced with repetitive stimulation, resulting in a larger release of quanta and larger end-plate potentials. This occurs with rapid stimulation because the frequency of stimulation is faster than the time it takes for calcium to leave the presynaptic terminal (100 to 200 ms), leading to higher levels of calcium influx and larger end-plate potentials. The incremental response has to be more than 50% to be considered diagnostic.

Needle EMG is usually normal in LEMS.

Katirji B. Electromyography in Clinical Practice: A Case Study Approach, 1st ed. St. Louis: Mosby; 1998.

Preston DC, Schapiro BE. Electromyography and Neuromuscular Disorders, 2nd ed. Philadelphia, PA: Elsevier; 2005.

10. b

There are different types of skeletal muscle fibers, which vary in their contractile speed, ATPase activity, and source of energy, among other characteristics. Muscle fibers type I are also called slow-oxidative, have slow ATPase activity and large oxidative capacity, with large numbers of mitochondria. They are red in color and small in diameter. Type IIa fibers are also called fast-oxidative-glycolytic, and these fibers have fast ATPase activity, with high glycolytic capacity and moderate oxidative capacity. These fibers are fast and resistant to fatigue. They are red in color and large in diameter. Type IIb fibers are also called fast-glycolytic, and these fibers have fast ATPase activity, with high glycolytic capacity but low oxidative capacity. These fibers are fast and fatigable. Their color is pale and diameter is large.

Barret KE, Barman SM, Boitano S, et al. Ganong’s Review of Medical Physiology, 23rd ed. New York: McGraw-Hill; 2010.

11. b, 12. a

This patient has chronic inflammatory demyelinating polyneuropathy (CIDP), which is a symmetric demyelinating polyneuropathy presenting with proximal and distal weakness with or without sensory loss and hypo- or areflexia. Autonomic involvement may occur, but it is less common than in Guillain-Barre syndrome (GBS). CIDP usually presents in adult patients between 40 and 60 years of age, and it is progressive and/or relapsing, with a time course of at least 8 weeks necessary for the diagnosis to be made.

In CIDP, CSF may demonstrate albuminocytologic dissociation. If the CSF cell count is elevated more than 10/mm3 (normal up to 5 lymphocytes/mm3), an alternative cause should be investigated.

In CIDP, electrophysiologic studies demonstrate features of a demyelinating neuropathy. Figure 9.2 shows right median motor NCS with stimulation at the wrist and at the elbow, with findings consistent with a demyelinating neuropathy. On NCS, demyelination is associated with marked slowing of conduction velocity (slower than 75% of the lower limit of normal), marked prolongation of distal latency (longer than 130% of the upper limit of normal), or both. In this case, the peak latency is prolonged (normal is <4 ms) and the conduction velocity is reduced (normal >50 m/s), with the presence of CMAP amplitude dispersion. All these features are consistent with demyelination. Although not present in this case, conduction block is common in CIDP. Sensory NCS do not play a major role in the diagnosis of CIDP.

Sural nerve biopsy should be considered when the CSF or electrophysiologic studies are not supportive of the diagnosis. In CIDP the biopsy will typically show evidence of demyelination and remyelination with onion bulb formation, and sometimes evidence of inflammation. Nerve biopsy may be helpful to exclude other conditions.

When the clinical symptoms do not progress beyond 4 weeks, GBS is the likely diagnosis. When symptoms relapse after treatment and/or symptom progression extends beyond 4 weeks (but <8 weeks), the diagnosis is controversial. Some authors suggest the term subacute inflammatory demyelinating polyradiculoneuropathy to describe patients in this time frame, with manifestations similar to both acute inflammatory demyelinating polyneuropathy and CIDP. CIDP is the diagnosis when symptoms progress or relapse beyond 8 weeks.

GBS is discussed in questions 15, 16, 24, 55, and 56. Multifocal motor neuropathy is discussed in questions 35 and 36. This patient does not have a disorder of the neuromuscular junction, and therefore this is not myasthenia gravis or Lambert-Eaton myasthenic syndrome (discussed in Chapter 10).

Saperstein DS. Chronic acquired demyelinating polyneuropathies. Semin Neurol. 2008; 28:168–184.

13. d

After 3 weeks from an injury, NCS cannot localize a focal axon loss lesion.

Peripheral nerve injury severity can range from focal demyelination to axonal injury and finally nerve transection with discontinuity of the nerve. Electrophysiologic studies can help determine the degree of injury.

A focal nerve injury can cause segmental demyelination, which is characterized by the presence of slowing at a specific site or the presence of a conduction block, which is a decrease in the CMAP amplitude with proximal stimulation as compared to distal stimulation, without significant temporal dispersion. The presence of conduction block therefore suggests segmental demyelination and helps localize the site of injury. A conduction block is reversible, given that the lesion is demyelinating.

If the injury is severe, an axon loss lesion may occur, eventually leading to wallerian degeneration, which is typically completed in 7 to 10 days from the injury. During this time, a conduction block may be observed due to axon loss. After 10 days, the distal axon degenerates and can no longer conduct. Therefore the conduction block due to axonal interruption resolves.

Once denervation occurs, spontaneous muscle activity appears on EMG, manifested by fibrillation potentials, which usually appear after the third week from the injury.

Katirji B. Electromyography in Clinical Practice: A Case Study Approach, 1st ed. St. Louis: Mosby; 1998.

Preston DC, Schapiro BE. Electromyography and Neuromuscular Disorders, 2nd ed. Philadelphia, PA: Elsevier; 2005.

14. c

This patient’s history is consistent with carpal tunnel syndrome, the most common entrapment neuropathy. The carpal tunnel is bounded dorsally and laterally by the carpal bones, and the transverse carpal ligament forms the palmar border. The structures that pass through the carpal tunnel include the median nerve most superficially (on the palmar aspect), flexor pollicis longus tendon, four tendons of the flexor digitorum superficialis, and four tendons of the flexor digitorum profundus.

Carpal tunnel syndrome may be unilateral or bilateral. Symptoms of carpal tunnel include pain and paresthesias in the medial half of the palm and first three digits and lateral half of the fourth digit that classically awaken the patient at night, with patients shaking their hand to relieve symptoms. Shooting pains may radiate up the forearm or even the upper arm; therefore, presence of forearm or upper arm symptoms should not exclude the diagnosis of carpal tunnel syndrome. Sensation is spared over the thenar eminence because the palmar sensory branch that innervates the thenar eminence travels outside the carpal tunnel. Physical examination may show provocation of symptoms with repetitive tapping on the median nerve at the wrist (Tinel’s sign) and with hyperflexion of the wrists for a period of time (Phalen’s maneuver). The only motor branch of the median nerve distal to the carpal tunnel is the thenar (or recurrent) motor branch; in advanced carpal tunnel, the thenar muscles are therefore weak and atrophied and show evidence of denervation on EMG. In such cases, there is weakness of thumb abduction and opposition, with weakness in activities requiring fine motor coordination of the first three digits such as buttoning.

Figure 9.3 shows prolonged distal median nerve SNAP latency. Motor NCS and EMG are normal, indicating mild carpal tunnel. With more advanced demyelination, evidence of conduction block at the wrist may be found, but it requires more distal stimulation near or at the palm for confirmation. With more severe carpal tunnel, with axon loss, prolonged distal motor latencies and reduced amplitudes of the CMAP and SNAP are seen. When severe, there is also evidence of denervation in median nerve–innervated thenar muscles on EMG.

Absence of neck pain makes radiculopathy less likely. In addition, the brachioradialis deep tendon reflex is intact. Although a C6 radiculopathy would lead to sensory loss over the lateral thumb, it would also lead to sensory loss over the lateral forearm, in addition to weakness in forearm flexion and supination, which are not evident in this case. A C7 radiculopathy leads to sensory loss over the third digit, but both a C6 and C7 radiculopathy are less likely on the basis of the abnormal median nerve sensory NCS, and absence of weakness or fibrillation potentials in C6- and C7-innervated muscles. In radiculopathy, because the dorsal root ganglion lies distal to the lesion, sensory NCS remain normal, despite sensory complaints the patient may have. A brachial plexopathy would usually not lead to the limited distribution of sensory symptoms without motor symptoms as well. Absence of weakness in more proximal median nerve–innervated muscles, and the presence of conduction block at the wrist, supports the lesion being at the level of the wrist rather than the elbow.

Guillain-Barre syndrome is discussed in questions 15, 16, 24, 55, and 56.

Preston DC, Shapiro BE. Electromyography and Neuromuscular Disorders. 2nd ed. Philadelphia, PA: Elsevier; 2005.

Russell SM. Examination of Peripheral Nerve Injuries, New York: Thieme; 2006.

15. a, 16. b

This patient has Guillain-Barre syndrome (GBS). This is an acute inflammatory demyelinating polyradiculoneuropathy and a common cause of ascending paralysis. It affects any age group and any gender in all parts of the world. In about 60% of cases, there is a preceding respiratory or gastrointestinal illness 1 to 3 weeks prior. Campylobacter jejuni is an identifiable etiology of the preceding gastrointestinal illness; however, it is the cause in only a limited number of cases and predominantly associated with the axonal variant.

GBS is considered to be caused by an immunologic reaction to the peripheral nerves. It is not clear what triggers the immunologic reaction, but it is thought that an infectious or environmental process may contribute to the process in susceptible individuals. The main mechanism is a T-cell-mediated response against myelin proteins that occurs after encountering a cross-reactive antigen, leading to the release of cytokines and activation of macrophages that will damage peripheral myelin. During the process, there is mononuclear infiltration into the peripheral nerves, T-lymphocyte activation, and antibody binding to Schwann cells and myelin components, with macrophages targeting the myelin components. Axonal variants of GBS exist, and are less responsive to treatment with protracted course and worse prognosis.

The typical clinical presentation begins with sensory symptoms, especially numbness and paresthesias beginning distally, usually in the toes and feet, ascending through the lower extremities, and later the upper extremities. Some patients complain of burning pain, with cramps or muscle discomfort. Lower extremity weakness begins later, and is usually symmetric and ascending from distal to proximal, evolving over days. There is subsequent involvement of the hands and upper limbs, and weakness of the respiratory and bulbar muscles may occur later. Reduced or absent reflexes are evident on examination.

Involvement of the autonomic nervous system and respiratory muscles may occur in patients with GBS, and there may be rapid progression with life-threatening complications, such as arrhythmias, bradycardia, tachycardia, hemodynamic instability, and respiratory failure. A patient who is rapidly worsening should be observed with cardiac monitoring and frequent vital signs, as well as frequent evaluation of respiratory parameters, including negative inspiratory force and vital capacity. A negative inspiratory force of less than −30 cc H2O or vital capacity of less than 15 to 20 mL/kg support elective endotracheal intubation.

The diagnosis of GBS is based on clinical history and examination, and monitoring respiratory function in a rapidly deteriorating patient with a characteristic history and examination is paramount. Other supporting tests are sometimes helpful to make the diagnosis and exclude other potential causes. Identifying cytoalbuminologic dissociation (i.e., high CSF protein and few WBCs) supports the diagnosis. EMG/NCS may be normal in the acute setting and will not likely affect the acute care of the patient. MRI of the lumbar spine may show gadolinium enhancement of the cauda equina, but this will not affect the treatment. MRI of the brain is not indicated.

Hughes RAC, Cornblath DR. Guillain-Barre syndrome. Lancet. 2005; 366:1653–1666.

Ropper AH, Samuels MA. Adams and Victor’s Principles of Neurology, 9th ed. New York: McGraw-Hill; 2009.

17. d

As discussed further below and shown in Figure 9.5, both the dorsal scapular nerve and the long thoracic nerve arise directly from the ventral (anterior) rami of the nerve roots.

FIGURE 9.5 Brachial plexus anatomy diagram (Illustration by David Schumick, BS, CMI. Reprinted with permission of the Cleveland Clinic Center for Medical Art & Photography. © 2010. All Rights Reserved)

The upper extremity receives innervation from the C5 to T1 nerve roots. In the intervertebral foramina, the motor and sensory roots join to form a spinal nerve, which then branches into ventral and dorsal rami before exiting the foramina. The ventral (anterior) rami of these nerve roots join to form a plexus of nerves known as the brachial plexus (Figure 9.5). The innervation of the upper extremities is discussed below.

-Roots

Two nerves that innervate the upper extremity branch off the nerve roots themselves. The dorsal scapular nerve, which innervates the rhomboids and levator scapulae, arises from the C5 nerve root. The long thoracic nerve, which innervates the serratus anterior, arises from the C5 to C7 roots.

-Trunks

The ventral rami of the C5 to T1 nerve roots join to form the trunks of the brachial plexus.

The upper (or superior) trunk, formed from the C5 and C6 nerve roots, gives off two branches: the suprascapular nerve, which innervates the supraspinatus and infraspinatus, and the nerve to subclavius. The point where the C5 and C6 nerve roots meet is called Erb’s point.

The middle trunk is formed from the C7 root. There are no branches from the middle trunk.

The lower (inferior trunk) is formed from the C8 and T1 roots. There are no branches from the inferior trunk.

The trunks then divide into anterior and posterior divisions.

-Cords and nerves

The cords are named according to their relationship to the axillary artery.

The lateral cord is formed from the anterior divisions of the superior and middle trunk, and therefore carries fibers from C5 to C7. The lateral cord gives rise to the lateral pectoral nerve, which innervates the pectoralis major. The lateral cord ends as two nerves, the median nerve (which also receives a contribution from the medial cord), discussed further in question 23, and the musculocutaneous nerve, which contains the lateral antebrachial cutaneous nerve and is further discussed in question 67.

The posterior cord is formed from the posterior divisions of the upper, middle, and lower trunk, and therefore carries fibers from C5 and C8. Three nerves arise from the posterior cord: (1) the upper subscapular nerve, which contains predominantly C7 and C8 fibers and innervates the subscapularis; (2) the lower subscapular nerve, which contains C5 and C6 fibers and innervates the teres major and the lower part of the subscapularis; and (3) the thoracodorsal nerve, which contains C6, C7, and C8 fibers and innervates the latissimus dorsi. The posterior cord ends as two nerves, the axillary nerve, which is discussed further in question 69, and the radial nerve, which is discussed further in question 43.

The medial cord is a continuation of the lower (inferior) trunk. The medial cord gives off three nerves: (1) the medial pectoral nerve, which contains predominantly C8 and T1 fibers and innervates the pectoralis minor; (2) the medial brachial cutaneous nerve, which provides sensory innervation to the medial arm; and (3) the medial antebrachial cutaneous nerve, which provides sensory innervation to the medial forearm. The medial cord gives fibers to the median nerve and then continues as the ulnar nerve, which is discussed further in question 37.

Russell SM. Examination of Peripheral Nerve Injuries. New York: Thieme; 2006.

18. a, 19. b

The patient in question 18 has familial amyloid polyneuropathy (FAP) type 1, as evidenced by an examination suggestive of a polyneuropathy, autonomic features, and a family history. The patient in question 19 has FAP2, as evidence by the presence of carpal tunnel, a family history of carpal tunnel, mild predominantly sensory polyneuropathy, and absence of prominent autonomic features.

The familial FAPs are a group of autosomal dominant multisystem disorders that result from deposition of amyloid proteins in the peripheral nerves and other organs including the heart and kidney.

The most common FAPs, types 1 and 2, result from various mutations in transthyretin, a plasma protein that is synthesized predominantly in the liver and transports thyroxine and other proteins.

FAP1 is characterized by onset of symptoms in the third to fourth decades of life. Both small and large nerve fibers are affected, but loss of pain and temperature sensation is most pronounced, with relative sparing of posterior column modalities. Symptoms include lancinating pains and dysesthesias, and autonomic dysfunction, including sexual dysfunction, orthostatic hypotension, urinary symptoms, gastrointestinal symptoms, anhidrosis, and pupillary abnormalities. Cardiac and renal involvement may occur from amyloid deposition in these organs. Nerve, rectal, or fat pad biopsy with congo red staining demonstrates amyloid, which exhibits an apple-green birefringence on polarized light; transthyretin molecular gene testing is also available for diagnosis. Liver transplant may be helpful for FAPs resulting from transthyretin mutations.

FAP2, as depicted in question 19, manifests later than FAP1, in the fourth and fifth decades, and its main features are carpal tunnel syndrome and a slowly progressive polyneuropathy, with absence of autonomic features.

FAP3 and -4 are rare and not related to abnormalities in transthyretin. FAP3 is similar to FAP1 in clinical manifestations, but with earlier renal involvement and more gastrointestinal involvement, with a higher incidence of duodenal ulcers. Other features may include hypothyroidism, adrenal insufficiency, and sexual dysfunction. It results from abnormalities in the apolipoprotein A1 gene. FAP4 manifests in the third decade of life, with corneal dystrophy being a prominent early feature. In later life, cranial neuropathies and skin changes occur; cranial nerves VII, VIII, and XII are commonly affected. Other features include a peripheral sensorimotor neuropathy and carpal tunnel syndrome, without autonomic signs or symptoms. FAP4 results from abnormalities in the amyloid protein gelsolin.

The FAPs are distinguished on the basis of family history and laboratory testing from primary systemic amyloidosis (which results from deposition of AL amyloid, and is associated with monoclonal protein), and secondary amyloidosis (which results from deposition of AA amyloid, and occurs in the setting of systemic inflammatory diseases).

Bradley WG, Daroff RB, Fenichel GM, et al. Neurology in Clinical Practice, 5th ed. Philadelphia, PA: Elsevier; 2008.

Ropper AH, Samuels MA. Adams and Victor’s Principles of Neurology, 9th ed. New York: McGraw-Hill; 2009.

20. d

The diarrhea that occurs from diabetic autonomic neuropathy typically occurs at night.

Autonomic dysfunction resulting from diabetic neuropathy can involve multiple organ systems. In the cardiovascular system, manifestations include resting tachycardia or bradycardia, loss of the respiratory variability of the heart rate, loss of the normal tachycardic response, orthostatic hypotension, and increased risk of silent myocardial infarction. Gastrointestinal abnormalities may occur, ranging from delayed gastric emptying, constipation from colonic atony, bacterial overgrowth, and diarrhea, which is typically nocturnal. Neurogenic bladder may occur, as well as sexual dysfunction caused by impotence, erectile dysfunction, and retrograde ejaculation. Abnormalities in sudomotor function also occur, with areas of anhidrosis and hyperhydrosis.

Bradley WG, Daroff RB, Fenichel GM, et al. Neurology in Clinical Practice, 5th ed. Philadelphia, PA: Elsevier; 2008.

Ropper AH, Samuels MA. Adams and Victor’s Principles of Neurology, 9th ed. New York: McGraw-Hill; 2009.

21. a, 22. d

This patient’s history and examination are consistent with Charcot-Marie-Tooth (CMT) 1. The clinical manifestations are discussed further below, but respiratory compromise is not usually seen in the most common forms of CMT, but occurs in some cases of CMT2C.

The CMTs, also known as hereditary sensorimotor neuropathies or peroneal muscular atrophy, are a large, heterogeneous group of inherited peripheral neuropathies. The CMTs can be divided into demyelinating, axonal, and combined demyelinating and axonal forms. They are genetically heterogeneous.

The demyelinating CMTs include CMT1 and CMTX. CMT4, discussed in question 39, involves both demyelination and axon loss. In the demyelinating CMTs, NCS generally show diffuse, uniformly slow conduction velocities without conduction blocks or temporal dispersion, indicating a hereditary as opposed to acquired demyelinating process (discussed in question 12). There may be evidence of axon loss in the demyelinating forms as well, particularly in patients with long-standing disease; this is secondary axon loss, and conduction velocities as well as CMAP and SNAP amplitudes will be reduced. In contrast, with the axonal forms, motor and sensory potential amplitudes will be reduced, but conduction velocities will be normal or minimally reduced (discussed in question 3, 4, and 13).

At the time of this publication, there were seven subtypes of CMT1; they are all inherited in an autosomal dominant fashion. CMT1A is the most common inherited demyelinating neuropathy. Clinical manifestations typically begin in the first two decades of life and include slowly progressive weakness, muscle atrophy, kyphosis, and mild (often asymptomatic) sensory loss. Other signs include hammertoes, high-arched feet, palpably enlarged nerves due to peripheral nerve hypertrophy (which more commonly occurs in the CMT1 group compared to the other CMTs), and pes cavus. Involvement of the upper extremities typically occurs later in life. There is often a family history of neuropathy, though due to variable expression, some affected family members may only have mild features such as hammertoes and may remain undiagnosed for a large part of their life; sporadic cases without a clear family history also exist. Other forms of CMT may be congenital and severe. In the CMT1 group, many of the genes involved are related to myelin synthesis. CMT1A is due to a duplication in the peripheral myelin protein 22 (PMP22) gene on chromosome 17, whereas CMT1B is due to a mutation in the myelin protein 0 gene. CMT1B is more severe in terms of clinical manifestations as compared to CMT1A. In patients with CMT1, CSF shows elevated protein levels in some cases. Roussy-Levy syndrome is phenotypically similar to CMT1A but is associated with the presence of a static tremor and gait ataxia. It has been associated with mutations in both the PMP22and myelin protein 0 genes.

On nerve biopsy, pathologic features of the demyelinating inherited polyneuropathies include demyelination and an onion-bulb appearance due to Schwann cell proliferation. Onion bulbs are not specific for CMT, and also occur with chronic inflammatory demyelinating polyneuropathy (discussed in questions 11, 12, and 38).

CMTX is the second most common type of CMT. It is demyelinating and is clinically similar to CMT1, but it is X-linked in inheritance; males therefore tend to be more severely affected compared to females. It is due to a mutation in the connexin 32 gene.

CMT2 accounts for approximately one-third of the autosomal dominant inherited neuropathies. The CMT2 group are axonal neuropathies; NCS show normal conduction velocities, and nerve biopsy shows axon loss without evidence of significant demyelination. Compared to CMT1, in CMT2, symptoms and signs typically appear later and foot and spine deformities are less severe. Clinical manifestations do not always help distinguish the different CMTs, but some clinical features occur more frequently in the different subtypes. Optic atrophy occurs more in CMT2A2, foot ulcerations in CMT2B, and vocal cord paralysis, intercostal, and diaphragmatic weakness in CMT2C. In CMT2D, unlike in the other CMTs, the hands are involved more than the feet. Peripheral nerve hypertrophy does not occur. They are all autosomal dominant in inheritance, except for a subtype of CMT2A. The genes implicated in the CMT2 group are involved in axonal transport and membrane trafficking. CMT2A2 is one of the most common of this group and is due to mutations in mitofusin 2.

CMT3, also known as Dejerine-Sottas syndrome or hypertrophic neuropathy of infancy, is one of the more severe forms of the demyelinating CMTs. It presents in infancy with proximal weakness, absent deep tendon reflexes, and hypertrophy of the peripheral nerves. Prominent sensory symptoms including pain and dysesthesias occur. Patients typically have extensive disability early in life. Both autosomal recessive and dominant forms exist. CSF protein is usually elevated. Congenital hypomyelination is seen along the spectrum of this disorder. CMT3 is genetically heterogeneous; mutations in several genes including PMP22, protein myelin 0, and other genes implicated in defective demyelination are associated with CMT3. CMT4 is discussed in question 39.

The CMTs can be misdiagnosed as muscular dystrophy; NCS findings, as well as the presence of hammertoes and high-arched foot, distinguish between the two.

Bradley WG, Daroff RB, Fenichel GM, et al. Neurology in Clinical Practice, 5th ed. Philadelphia, PA: Elsevier; 2008.

Ropper AH, Samuels MA. Adams and Victor’s Principles of Neurology, 9th ed. New York: McGraw-Hill; 2009.

23. e

If the upper extremity were to be divided into upper arm (above the elbow), forearm (elbow to wrist), and hand (below the wrist), the median nerve does not innervate any muscles in the upper arm.

The median nerve is derived from the lateral and medial cords. The median nerve runs down the midline of the arm and crosses over the brachial artery to lie just medial to it as it passes under the bicipital aponeurosis in the antecubital fossa. In the forearm, the median nerve innervates pronator teres, flexor carpi radialis, and flexor digitorum superficialis (see Table 9.1 for root innervations and action of the muscles innervated by the median nerve).

TABLE 9.1 Upper extremity muscles innervated by the median nerve, their root innervation, and action

In the forearm, the median nerve gives off the anterior interosseus nerve that innervates flexor digitorum profundus to the second and third digits, flexor pollicis longus, and pronator quadratus. Before entering the carpal tunnel, the median nerve gives off the palmar cutaneous sensory nerve, a pure sensory nerve. The median nerve then passes through the carpal tunnel (discussed in question 14) and gives off the thenar motor branch, which innervates abductor pollicis brevis and opponens pollicis. The median nerve also innervates the first and second lumbricals.

The median nerve provides sensory innervations to the lateral (radial) two-thirds of the palm and the distal dorsal aspect of the first to third digits and the distal lateral (radial) half of the fourth digit through the palmar cutaneous nerve and through digital branches.

The median nerve is prone to injury with supracondylar fractures. Median nerve palsy can also occur due to entrapment in ligaments or between muscles (discussed in question 59).

Russell SM. Examination of Peripheral Nerve Injuries. New York: Thieme; 2006.

24. e

Intravenous immunoglobulins (IVIG) and plamapheresis both are used for the treatment of Guillain-Barre syndrome (GBS). Plasmapheresis (plasma exchange) has been evaluated in clinical trials, demonstrating benefit within the first 4 weeks from symptom onset and a trend toward additional benefit when administered earlier. Benefits include reduction of the duration of hospitalization, need for mechanical ventilation, more improvement on disability scales, and shortened time to walking unaided. Plasmapheresis is used in four to six treatments of 200 to 250 mL/kg. IVIG has been tested as well, and its use has shown similar efficacy to plasmapheresis. Its dose is 400 mg/kg/day for 5 days. The combination of plasmapheresis and IVIG has failed to show additional improvement when compared to either therapy alone. The use of steroids is not beneficial and not recommended. Pyridostigmine is used for symptomatic treatment of myasthenia gravis (discussed in Chapter 10), and not for GBS.

Hughes RAC, Cornblath DR. Guillain-Barre syndrome. Lancet. 2005; 366:1653–1666.

Hughes RAC, Widjicks EFM, Barohn R, et al. Practice parameter: Immunotherapy for Guillain-Barre syndrome: Report of the Quality Standards Subcommittee of the American Academy of Neurology. Neurology. 2003; 61:736–740.

Ropper AH, Samuels MA. Adams and Victor’s Principles of Neurology, 9th ed. New York: McGraw-Hill; 2009.

25. c, 26. d

The iliohypogastric, ilioinguinal, and genitofemoral nerves do not arise from the lumbosacral trunk, but rather from the lumbar plexus. The tensor fascia latae abducts the thigh when the hip is flexed, not extended.

The lumbosacral plexus consists of the lumbar and sacral plexus, connected via the lumbosacral trunk (Figure 9.6).

FIGURE 9.6 Lumbar plexus anatomy diagram (Illustration by David Schumick, BS, CMI. Reprinted with permission of the Cleveland Clinic Center for Medical Art & Photography. © 2010. All Rights Reserved)

The lumbar plexus is formed by contributions from T12 to L4 and gives rise to three major and three minor nerves. The three minor nerves are the iliohypogastric, ilioinguinal, and genitofemoral. The first two arise from a common trunk originating from L1 with some contributions from T12. The genitofemoral nerve arises from L1 and L2. The three major nerves are the femoral, obturator, and lateral femoral cutaneous. The lateral femoral cutaneous nerve originates from L2 and L3 (discussed in question 47). The femoral nerve originates from the posterior divisions of L2, L3, and L4 (see question 29 and Figure 9.7). The obturator nerve originates from the anterior divisions of L2, L3, and L4 and divides into an anterior and a posterior division. The anterior division gives innervation to the adductor brevis, adductor longus, and gracilis muscles. The posterior division gives innervation to the obturator externus and a portion of the adductor magnus, which is also innervated by the sciatic nerve.

The lumbosacral trunk is a structure that originates from L4 and L5 and joins the sacral plexus to form the sciatic nerve, which is not only the largest nerve of the lumbosacral plexus, but the largest nerve in the body.

The sacral plexus originates from the L4, L5, S1, S2, S3, and S4 nerve roots, with L4 and L5 provided by the lumbosacral trunk as already described. The anterior divisions of L4 through S3 contribute to form the tibial division of the sciatic nerve. The posterior divisions from L4 through S2 contribute to the common peroneal division of the sciatic nerve.

The superior gluteal nerve originates from L4, L5, and S1 and innervates the gluteus medius, gluteus minimus, and tensor fascia latae. These muscles contribute to thigh abduction, with the tensor fascia latae acting as the main abductor when the hip is flexed, and the gluteus medius and minimus acting as the main abductors when the hip is extended. The inferior gluteal nerve originates from L5, S1, and S2 and innervates the gluteus maximus, which is an extensor of the thigh. The posterior cutaneous nerve of the thigh originates from S1, S2, and S3 and gives sensory cutaneous innervation to the lower buttock and posterior thigh. The pudendal nerve originates from S2, S3, and S4 and provides sensory innervation to the perineal region and perianal region through the inferior rectal nerve, perineal nerve, and dorsal nerve of the penis or clitoris (discussed in Chapter 10).

Katirji B. Electromyography in Clinical Practice: A Case Study Approach, 1st ed. St. Louis: Mosby; 1998.

Russell SM. Examination of Peripheral Nerve Injuries. New York: Thieme; 2006.

27. d

This patient has hereditary neuropathy with liability to pressure palsies (HNPP). It is an autosomal dominant predominantly demyelinating hereditary neuropathy with incomplete penetrance and is caused by a deletion in the peripheral myelin protein 22 gene. Duplications in this same gene are the cause of Charcot-Marie-Tooth type (CMT) 1A. Patients with HNPPs classically present with recurrent episodes of focal mononeuropathies or plexopathies of the upper or lower limbs; the peroneal nerve is most commonly affected, followed by the ulnar nerve. The presentation is typically in young adulthood, and a history of compression or traction on the involved nerve can often be elicited. The weakness is not preceded or accompanied by pain, distinguishing HNPP from hereditary neuralgic amyotrophy, which can also lead to recurrent upper extremity mononeuropathies. Other phenotypes of HNPP, including one resembling CMT or a chronic sensorimotor demyelinating polyneuropathy resembling chronic inflammatory demyelinating polyneuropathy, also exist.

NCS show prolonged distal latencies, focal slowing at sites of compression, and in some cases diffuse reductions in SNAP amplitudes. Nerve biopsy shows a characteristic pattern of focal, sausage-like areas of thickening in the myelin called tomacula, as well as evidence of segmental demyelination and axon loss.

Bradley WG, Daroff RB, Fenichel GM, et al. Neurology in Clinical Practice, 5th ed. Philadelphia, PA: Elsevier; 2008.

28. a

The sciatic nerve originates from the L4, L5, S1, S2, and S3 roots. This nerve is the largest nerve in the body, and gives off two initial branches: the superior and inferior gluteal nerves. The superior gluteal nerve innervates the gluteus medius, minimus, and tensor fascia latae. The inferior gluteal nerve innervates the gluteus maximus (see Table 9.2).

TABLE 9.2 Lower extremity muscles innervated by superior and inferior gluteal nerves, their root innervation, and action

The sciatic nerve is composed of two different nerves running together: the tibial nerve medially and the common peroneal nerve laterally. In the thigh, the tibial division innervates the adductor magnus, semimembranosus, semitendinosus, and long head of the biceps femoris. The short head of the biceps femoris is supplied by the common peroneal division. The tibial nerve then continues in the posterior aspect of the leg and gives innervation to the gastrocnemius, soleus, and tibialis posterior. The peroneal nerve continues, and after the popliteal fossa, it passes behind the fibular head and divides into the superficial and deep peroneal nerves. The superficial peroneal nerve gives off branches to the peroneus longus and brevis, which permit foot eversion. The deep peroneal nerve supplies the tibialis anterior, extensor hallucis, extensor digitorum longus and brevis, and peroneus tertius. A lesion in the deep peroneal nerve produces foot drop with inability to dorsiflex the foot without impairing eversion of the foot. Preservation of foot inversion distinguishes peroneal neuropathy from L5 radiculopathy, in which the tibialis posterior muscle (innervated by the tibial nerve) is involved, impairing foot inversion.

Katirji B. Electromyography in Clinical Practice: A Case Study Approach, 1st ed. St. Louis: Mosby; 1998.

Russell SM. Examination of Peripheral Nerve Injuries. New York: Thieme; 2006.

29. a

The femoral nerve (Figure 9.7) is a large nerve that originates from the posterior divisions of L2, L3, and L4, traveling through the psoas major muscle which it innervates, then passing through the iliacus muscle which it also innervates. After this course, the femoral nerve passes under

FIGURE 9.7 Femoral nerve anatomy diagram (Illustration by David Schumick, BS, CMI. Reprinted with permission of the Cleveland Clinic Center for Medical Art & Photography. © 2010. All Rights Reserved)

the inguinal canal into the femoral triangle, located lateral to the femoral artery. It then divides into several terminal branches. There are three cutaneous branches: (1) the medial femoral cutaneous, (2) intermediate femoral cutaneous, and (3) saphenous nerve.

These branches carry sensory information from the anteromedial thigh, medial leg, medial malleolus, and arch of the foot. The motor branches provide innervation to the quadriceps (rectus femoris, vastus lateralis, medialis, and intermedius), sartorius, and pectineus (see Table 9.3). The patellar reflex is carried through the femoral nerve.

TABLE 9.3 Lower extremity muscles innervated by the femoral and obturator nerves, their root innervation, and action

Femoral nerve injury will manifest as weakness in hip flexion and knee extension, loss of the patellar reflex and sensory findings in the anteromedial thigh and medial leg. The femoral nerve can be injured in the retroperitoneal or intrapelvic space, or at the inguinal ligament. Clinically, the distinction between injury at these sites can be made by detection of weakness on hip flexion that will represent psoas compromise and electrophysiologically by the presence of fibrillations in the iliacus muscle. Both these muscles are innervated before the inguinal ligament, and their compromise will suggest an intrapelvic injury rather than an inguinal injury.

At the inguinal region, the femoral nerve can be damaged by inguinal masses or hematomas, during hip surgery or perineal surgeries, especially associated with prolonged lithotomy position, such as in this case.

It is important to distinguish femoral nerve injury from L2-L3-L4 radiculopathy and lumbar plexopathy. The presence of impairment of other nerves will suggest these possible diagnoses. For example, adductor weakness suggests involvement of the obturator nerve, which can occur in L2-L3-L4 radiculopathy or a lumbar plexopathy. Also, the presence of weakness in the distal lower extremity muscles will imply injury to other nerves, excluding a selective femoral nerve injury. Abnormal SNAPs do not correlate with a radiculopathy from an intraspinal canal lesion, because SNAPs will be normal in these lesions (discussed in questions 3, 4, and 6).

Katirji B. Electromyography in Clinical Practice: A Case Study Approach, 1st ed. St. Louis: Mosby; 1998.

Russell SM. Examination of Peripheral Nerve Injuries. New York: Thieme; 2006.

30. d

This patient’s history and examination are consistent with Tangier’s disease. Tangier’s disease is a very rare autosomal recessive disorder. Neurologic manifestations include a symmetric predominantly sensory neuropathy with dissociated sensory loss (loss of pain and temperature with relative preservation of posterior column modalities), mimicking syringomyelia. In other patients, relapsing multifocal mononeuropathies may occur. It results from mutations in an adenosine triphosphate transporter protein that results in low-serum cholesterol levels and elevated triglyceride levels. Deposition of triglycerides occurs in the reticuloendothelial system, and bone marrow biopsy may show fat-laden macrophages; deposition of triglycerides in the tonsils accounts for the orange appearance of the tonsils in these cases.

Bradley WG, Daroff RB, Fenichel GM, et al. Neurology in Clinical Practice, 5th ed. Philadelphia, PA: Elsevier; 2008.

Ropper AH, Samuels MA. Adams and Victor’s Principles of Neurology, 9th ed. New York: McGraw-Hill; 2009.

31. e

This patient has a sensory neuronopathy and possibly a peripheral neuropathy in the setting of small cell lung cancer, and most likely this is a paraneoplastic syndrome. Paraneoplastic syndromes are manifestations of malignancy, and commonly manifest with neurologic involvement. Small cell lung cancer is the most common neoplasm associated with neurologic paraneoplastic syndromes; however, other malignancies can also present this way, including breast, ovarian, kidney, and prostate cancers, thymomas, and lymphomas.

Certain antibodies have been found to correlate with characteristic clinical presentations, and it is thought that these antibodies are directed against the nervous system, either peripheral or central. Anti-Hu (also known as ANNA-1) antibody is typically associated with peripheral neuropathy and sensory neuronopathy in the setting of small cell lung cancer, and will most likely be positive in this case, though anti-Hu antibody has been associated with a variety of other neurologic manifestations as well. Anti-voltage-gated calcium channel is present in Lambert-Eaton myasthenic syndrome in association with small cell lung cancer (discussed in Chapter 10). Anti-Yo is present in ovarian carcinoma and other malignancies, and manifests with cerebellar degeneration. Anti-Ri is associated with opsoclonus-myoclonus with or without ataxia in the setting of neoplasms of the lung or breast. Anti-MAG antibodies are antibodies against myelin-associated glycoprotein and are associated with demyelinating neuropathy in the setting of monoclonal gammopathy of unknown significance (discussed in Chap- ter 16).

Bradley WG, Daroff RB, Fenichel GM, et al. Neurology in Clinical Practice, 5th ed. Philadelphia, PA: Elsevier; 2008.

Darnell RB, Poster JB. Paraneoplastic syndromes involving the nervous system. N Engl J Med. 2003; 349:1543–1554.

Ropper AH, Samuels MA. Adams and Victor’s Principles of Neurology, 9th ed. New York: McGraw-Hill; 2009.

32. d

The sciatic nerve (Figure 9.8) arises from the L4, L5, S1, S2, and S3 nerve roots. It is composed of two nerves: (1) the tibial nerve located medially and (2) the common peroneal nerve located laterally (Figure 9.9). Both are in the same sheath, but remain separated throughout their course with no intercrossing fibers. The common peroneal nerve is more prone to injuries, because it is smaller, more lateral, and has less supportive tissue.

FIGURE 9.8 Sciatic nerve anatomy diagram (Illustration by David Schumick, BS, CMI. Reprinted with permission of the Cleveland Clinic Center for Medical Art & Photography. © 2010. All Rights Reserved)

FIGURE 9.9 Peroneal nerve anatomy diagram (Illustration by David Schumick, BS, CMI. Reprinted with permission of the Cleveland Clinic Center for Medical Art & Photography. © 2010. All Rights Reserved)

The sciatic nerve exits the pelvis through the sciatic notch, gives off the superior gluteal nerve, and then passes under the piriformis muscle, after which it gives off the inferior gluteal nerve. In the thigh, the tibial division of the sciatic nerve gives innervation to the semitendinosus, semimembranosus, long head of the biceps femoris, and adductor magnus. The peroneal division of the sciatic nerve gives a branch to the short head of the biceps femoris in the thigh. Proximal to the popliteal fossa, the sciatic nerve bifurcates into the tibial and common peroneal nerves. Abnormalities in the short head of the biceps femoris help distinguish a sciatic nerve lesion from a common peroneal lesion at the fibular head, as the short head of the biceps femoris is spared in the latter.

The tibial nerve gives off the sural nerve that provides sensory innervation to the lateral aspect of the leg and foot. The tibial nerve then continues down the leg, innervating the gastrocnemius, soleus, tibialis posterior, flexor digitorum longus, and flexor hallucis longus. At the medial ankle, the tibial nerve passes through the tarsal tunnel and gives three terminal branches: (1) the calcaneal, (2) medial plantar, and (3) lateral plantar nerves.

The common peroneal nerve runs around the fibular head from posterior to anterior and divides into the superficial and deep peroneal nerves. The superficial peroneal innervates the peroneus longus and brevis, as well as the skin in the lower two-thirds of the lateral aspect of the leg and dorsum of the foot. The deep peroneal innervates the tibialis anterior, extensor hallucis, extensor digitorum longus and brevis, and peroneus tertius (see Table 9.4). The sensory territory of the deep peroneal nerve is the web space between the first and second toes.

TABLE 9.4 Lower extremity muscles innervated by the tibial and peroneal nerves, their root innervation, and action

In this patient, both tibial and peroneal nerve–innervated muscles are affected, including the short head of the biceps femoris, suggesting a sciatic nerve lesion, rather than individual tibial or peroneal nerve involvement. The fact that the sural and superficial peroneal SNAPs are affected excludes the possibility of a radiculopathy. Finally, the conspicuous absence of gluteal nerve–innervated muscles makes the diagnosis of a lumbosacral plexopathy unlikely. The final diagnosis is thus a sciatic mononeuropathy.

Common and deep peroneal neuropathies are discussed in question 44. L5 radiculopathy is discussed in question 86. Tibial nerve injury is discussed in question 34.

Katirji B. Electromyography in Clinical Practice: A Case Study Approach, 1st ed. St. Louis: Mosby; 1998.

Russell SM. Examination of Peripheral Nerve Injuries. New York: Thieme; 2006.

33. d

There is evidence supporting the efficacy of steroids, plasmapheresis, and/or intravenous immunoglobulin (IVIG) for chronic inflammatory demyelinating polyneuropathy (CIDP, discussed in questions 11, 12, and 38). Natalizumab is used for the treatment of multiple sclerosis, not CIDP.

Prednisone can be used for initial therapy from doses of 1 mg/kg/day up to 100 mg/day. Once improvement is noticed, the dose can be slowly tapered to a maintenance dose. A steroid-sparing agent may be required, such as azathioprine, cyclosporine, methotrexate, or mycophenolate mofetil. IVIG can be used with a dose of 2 g/kg over 2 to 5 days and repeated every 4 to 6 weeks according to the response. Plasmapheresis can be used with two to three treatments per week for a total of five to ten treatments. The benefits, however, are transient, and may last from 3 to 8 weeks, requiring repeated treatments. Plasmapheresis may be effective in patients who have significant weakness and those who relapse on steroids, in which case the dose of steroids should be increased along with the frequency and number of plasmapheresis treatments.

Ropper AH, Samuels MA. Adams and Victor’s Principles of Neurology, 9th ed. New York: McGraw-Hill; 2009.

Saperstein DS. Chronic acquired demyelinating polyneuropathies. Semin Neurol. 2008; 28:168–184.

34. e

The tibial nerve is a division of the sciatic nerve, and at the level of the thigh, it provides innervation to the semimembranosus, semitendinosus, and long head of the biceps femoris (see Figure 9.8). Proximal to the popliteal fossa, the tibial division of the sciatic nerve separates from the peroneal division and gives off the sural nerve, which provides sensory innervation to the lateral aspect of the leg and foot. The tibial nerve then continues down the leg innervating the gastrocnemius, soleus, tibialis posterior, flexor digitorum longus, and flexor hallucis longus. The tibialis anterior is an L5 deep peroneal nerve–innervated muscle.

At the medial ankle, the tibial nerve passes under the flexor retinaculum through the tarsal tunnel and gives three terminal branches: (1) calcaneal, (2) medial plantar, and (3) lateral plantar. The calcaneal branch is purely sensory and innervates the heel. The medial plantar branch innervates the abductor hallucis, flexor digitorum brevis, and flexor hallucis brevis, as well as the skin of the medial sole. The lateral plantar branch innervates the abductor digiti quinti pedis, flexor digiti quinti pedis, adductor hallucis, and interossei, as well as the skin of the lateral sole.

An entrapment neuropathy of the tibial nerve at the level of the tarsal tunnel will not produce weakness on plantarflexion. This entrapment neuropathy may manifest with burning pain in the plantar region, worse with standing and walking, with sensory deficits in the sole and sometimes atrophy in this area. Sensation in the dorsum of the foot is normal, as well as the ankle reflex.

Katirji B. Electromyography in Clinical Practice: A Case Study Approach, 1st ed. St. Louis: Mosby; 1998.

Russell SM. Examination of Peripheral Nerve Injuries. New York: Thieme; 2006.

35. c, 36. d

This patient has multifocal motor neuropathy (MMN), also known as multifocal motor neuropathy with conduction block. This condition is a purely motor demyelinating neuropathy that presents with asymmetric weakness from involvement of individual peripheral nerves, hypo- or areflexia in the distribution of affected nerves, and no sensory manifestations. CSF studies show normal protein levels, in contrast to acute and chronic demyelinating polyneuropathies. Anti-GM1 antibodies have been detected in this condition, but the presence of these antibodies is not required to make the diagnosis, and does not predict response to therapy. Electrophysiologic testing demonstrates typical conduction block in various nerve distributions. The presence of conduction block is not however required to make the diagnosis if there are other features of demyelination, and the response to treatment is not different between patients with or without conduction block. Sensory NCS are normal.

Patients with MMN do not have a good response to steroids or plasmapheresis, and some may worsen with these therapies. The use of intravenous immunoglobulin has shown to be of benefit and is associated with clinical improvement. Other therapies that have been reported to be beneficial are rituximab and cyclophosphamide.

Chronic inflammatory demyelinating polyneuropathy (CIDP) is discussed in questions 11, 12, and 38. Acute inflammatory demyelinating polyneuropathy (AIDP) is discussed in questions 15, 16, 24, 55, and 56. Subacute inflammatory demyelinating polyneuropathy is a controversial diagnosis that is not generally accepted, but some authors have used this term to define patients with manifestations similar to AIDP and CIDP, with a time evolution between 4 and 8 weeks. Multifocal acquired demyelinating sensory and motor neuropathy is discussed in question 65.

Saperstein DS. Chronic acquired demyelinating polyneuropathies. Semin Neurol. 2008; 28:168–184.

37. b

The ulnar nerve is a continuation of the medial cord. The medial cord gives a contribution to the median nerve and then gives off two branches: (1) the medial brachial cutaneous nerve and (2) medial antebrachial cutaneous nerve, which provide sensory innervation to the medial half of the arm and forearm, respectively. It then continues as the ulnar nerve. The ulnar nerve predominantly carries C8 and T1 fibers. If the upper extremity were to be divided into upper arm (above the elbow), forearm (elbow to wrist), and hand (below the wrist), the ulnar nerve does not innervate any muscles in the arm.

At the elbow, it emerges from the triceps and enters the post-condylar groove, a bony canal between the medial epicondyle and the olecranon of the ulna. This is where the ulnar nerve is most susceptible to injury (see questions 41 and 45). The ulnar nerve then travels down the forearm, where it gives branches to flexor carpi ulnaris and then flexor digitorum profundus to the fourth and fifth digits (see Table 9.5 for root innervations and action of the muscles innervated by the ulnar nerve). It gives off two sensory branches: (1) the dorsal ulnar cutaneous nerve and (2) palmar ulnar cutaneous nerve. Proximal to the wrist it is joined by the ulnar artery.

TABLE 9.5 Upper extremity muscles innervated by the ulnar nerve, their root innervation, and action

The ulnar nerve then enters the hand via Guyon’s canal, which is bounded by the carpal bones dorsally and laterally, the transverse carpal ligament medially, and the palmar carpal ligament ventrally. The ulnar nerve then bifurcates into a deep motor branch and a superficial sensory branch distal to Guyon’s tunnel. The deep motor branch innervates hypothenar eminence muscles: abductor digiti minimi, flexor digiti minimi, and opponens digiti minimi. The ulnar nerve provides motor innervation to many of the intrinsic hand muscles, which are largely involved in fine finger movements: the fourth and fifth lumbricals, and the dorsal and palmar interossei. It also innervates two thenar muscles: (1) adductor pollicis and (2) flexor pollicis brevis.

The ulnar nerve provides sensory innervation to the hypothenar eminence, the palmar and dorsal medial portion of the hand, fifth digit, and half of the fourth digit.

Russell SM. Examination of Peripheral Nerve Injuries. New York: Thieme; 2006.

38. d

This patient is not suspected of having a neuromuscular junction disorder such as myasthenia gravis, and therefore acetylcholine receptor antibodies are not helpful. This patient has clinical features to suggest a chronic inflammatory demyelinating polyneuropathy (CIDP). In cases of CIDP in which the CSF shows a WBC count of more than 10/mm3, an underlying cause should be ruled out and tests should be obtained to investigate other etiologies, including HIV, hepatitis C, lymphoproliferative or myeloproliferative disorders, Lyme disease, and neurosarcoidosis. Monoclonal protein analysis should be obtained to evaluate for a serum paraprotein. EMG and NCS are part of the work-up of CIDP, and will show features of demyelination with conduction block.

Saperstein DS. Chronic acquired demyelinating polyneuropathies. Semin Neurol. 2008; 28:168–184.

39. c

By convention, recessive forms of Charcot-Marie-Tooth (CMT) are classified as CMT4. The CMT4 group consists of eight subtypes, which include both demyelinating and axon loss forms. These disorders are rare. They have a young age of onset, in early childhood, with significant disability. Clinical features of some of these subtypes include vision loss, severe scoliosis, and hearing loss.

The hereditary sensory and autonomic neuropathies are also autosomal dominant and discussed in questions 75 to 77. Hereditary neuropathy with liability to pressure palsy is discussed in question 27, and the familial amyloid polyneuropathies are discussed in questions 18 and 19.

Preston DC, Shapiro BE. Electromyography and Neuromuscular Disorders, 2nd ed. Philadelphia, PA: Elsevier; 2005.

40. d

Lumbosacral radiculopathy is commonly caused by disc herniation or degenerative spine changes. S1 radiculopathy commonly manifests as pain radiating from the buttock down the posterior thigh, posterior leg, and lateral foot, with sensory impairment in this dermatomal region, especially the lateral foot and fifth toe. The most prominent weakness is plantarflexion and toe flexion, and the ankle deep tendon reflex will be reduced or absent. Muscles involved in S1 radiculopathies include the abductor hallucis, abductor digiti quinti pedis, soleus, medial and lateral gastrocnemius, extensor digitorum brevis, biceps femoris (long and short head), and gluteus maximus. Muscles partially innervated by S1 that may also be affected are the tibialis posterior, flexor digitorum brevis, gluteus medius, and tensor fascia latae. Although the SNAPs should be normal in S1 radiculopathies, the H-reflex is commonly reduced or absent.

L2-L3-L4 radiculopathy is discussed in question 51. L5 radiculopathy is discussed in question 86. Peroneal neuropathy is discussed in question 44.

Katirji B. Electromyography in Clinical Practice: A Case Study Approach, 1st ed. St. Louis: Mosby; 1998.

Russell SM. Examination of Peripheral Nerve Injuries. New York: Thieme; 2006.

41. e

This patient’s history and examination are consistent with an ulnar neuropathy at or above the elbow. This can occur with trauma, lacerations, or blunt injury. There is significant loss of fine motor coordination due to weakness of the third and fourth lumbricals and the palmar and dorsal interossei. Flexor pollicis brevis is innervated by both the median and ulnar nerve, so even with complete ulnar palsy, some thumb abduction can still be achieved by the part of the muscle innervated by the median nerve. When asked to make a fist, the hand assumes the appearance of a claw, with the fourth and fifth digits hyperextended at the metacarpophalangeal joint and partially flexed at the interphalangeal joint. This occurs because the third and fourth lumbricals as well as the interossei and flexor digiti minimi are weak, and there is unopposed action of the radial nerve–innervated muscles (see Table 9.6), causing hyperextension at the metacarpophalangeal joints. Other signs seen with ulnar nerve palsy include (1) Wartenberg’s sign, or fifth digit abduction at rest due to paralysis of the third palmar interossei with unopposed action of extensor digiti minimi and extensor digitorum communis (radial nerve–innervated muscles), and (2) Froment’s sign, whereby during attempted forceful adduction of the thumb, as with an attempt to hold a piece of paper between the thumb and the index finger, thumb flexion occurs. This occurs because the adductor pollicis is weak, and thumb flexion (by the intact flexor pollicis longus) substitutes with thumb flexion. In the case described, reduction in CMAP amplitude of more than 50% with stimulation above as compared to below the elbow (Figure 9.4) further supports the diagnosis.

In patients with ulnar nerve compression at the wrist, at Guyon’s canal (as occurs in bicycle riders or others who frequently place pressure at the medial wrist area), claw hand, Wartenberg’s sign, and Froment’s sign can be seen. Involvement of flexor carpi ulnaris and flexor digitorum profundus (which receive motor branches from the ulnar nerve in the forearm) indicate that the lesion is proximal to the wrist. In addition, with lesions at or distal to the wrist, sensation over the hypothenar eminence is spared because the palmar cutaneous branch arises proximal to Guyon’s canal. With a lesion at the wrist, CMAP amplitudes would be abnormally low with stimulation at the wrist, and a reduction in CMAP amplitude would not occur with more proximal stimulation.

Other C8-innervated muscles such as the flexor digitorum profundus to the second digit are of normal strength, and forearm pronation is normal, providing evidence that this is not a C7 or C8 radiculopathy. Intact sensation proximal to the wrist excludes a proximal medial cord lesion; when sensory loss extends 2 cm or more proximal to the wrist, the territory innervated by the medial antebrachial cutaneous nerve (which branches directly off the medial cord), involvement of the medial cord should be suspected. Another feature distinguishing an ulnar nerve palsy from a medial cord lesion is intact strength of median nerve-innervated C8 and T1 muscles, such as flexor pollicis brevis, abductor pollicis brevis, opponens pollicis, and the median nerve–innervated lumbricals (see Table 9.1).

Russell SM. Examination of Peripheral Nerve Injuries. New York: Thieme; 2006.

42. e

This patient has diabetic amyotrophy, which is a polyradiculoneuropathy. These patients are usually older, and the beginning of symptoms may occur in the setting of mild diabetes; however, the onset is frequently associated with a period of transition, such as during the initiation or adjustment of insulin treatment, or associated with episodes of hypo- or hyperglycemia. The presentation begins with pain starting in the low back or hip and radiating down to the lower limb. Deep pain with superimposed lancinating sensation is described, and commonly more severe at night. The sensory deficit may involve the L2, L3, and L4 distributions. Days to weeks later, these patients develop weakness and eventually atrophy, which involve the pelvic girdle and thigh muscles. Muscles most frequently involved are the iliopsoas, glutei, thigh adductors, quadriceps, hamstrings, and sometimes the anterior tibial muscles. Patellar reflex is absent. The EMG shows evidence of denervation in the involved myotomes.

The progression is gradual and may continue for months. The pain often resolves spontaneously, and subsequently, there may be recovery of strength, but this may take many months and sometimes years. Contralateral involvement may occur in some cases, appearing months or years later.

This is not a diabetic mononeuropathy. Small-fiber diabetic neuropathy is discussed in question 49. Large-fiber diabetic neuropathy is discussed in question 85. Diabetic autonomic neuropathy is discussed in question 20.

Bradley WG, Daroff RB, Fenichel GM, et al. Neurology in Clinical Practice, 5th ed. Philadelphia, PA: Elsevier; 2008.

Ropper AH, Samuels MA. Adams and Victor’s Principles of Neurology, 9th ed. New York: McGraw-Hill; 2009.

43. e

One of the forearm muscles innervated by the radial nerve, the brachioradialis, is a forearm flexor.

The radial nerve is a continuation of the posterior cord. It carries C5, C6, C7, and C8 fibers. The first branch of the radial nerve is the posterior cutaneous nerve to the arm. In the arm, the radial nerve gives branches to long, medial, and lateral heads of the triceps muscle and then travels along the spiral groove (see Table 9.6 for root innervations and action of the muscles innervated by the radial nerve). It then gives off the posterior cutaneous nerve, which runs with the radial nerve in the spiral groove of the humerus. In the spiral groove, the radial nerve gives off the lateral cutaneous nerve to the arm. Distal to the spiral groove, it gives branches to the brachioradialis, and extensor carpi radialis longus and brevis. Distal to the elbow, it bifurcates into the posterior interosseus and superficial sensory radial nerves. The posterior interosseus nerve innervates extensor carpi ulnaris, extensor digitorum communis, extensor digiti minimi, abductor pollicis longus, extensor pollicis longus and brevis, and extensor indices.

Table 9.6 Upper extremity muscles innervated by the radial nerve, their root innervation, and action

The radial nerve provides sensory innervation to the posterior arm through the posterior cutaneous nerve to the arm, to the lateral arm via the lateral cutaneous nerve to the arm, and to the posterior forearm through the posterior cutaneous nerve to the forearm. The superficial sensory radial nerve provides sensory innervation to the dorsolateral half of the hand, the proximal two-thirds of the thumb (including the lateral thumb, over the anatomic snuffbox), and the second and third digits (discussed in question 66). Remember that the distal dorsal aspects of the second and third digits are innervated by the median nerve.

Russell SM. Examination of Peripheral Nerve Injuries. New York: Thieme; 2006.

44. a

This patient has a common peroneal nerve injury, likely localized to the fibular head. Peroneal mononeuropathy at the fibular head has been associated with significant weight loss, prolonged bedrest or hospitalization, leg crossing, diabetes, and other compressive mechanisms in this area.

The common peroneal nerve is a division of the sciatic nerve, giving off the lateral cutaneous nerve of the calf before turning around the fibular head and passing through the fibular tunnel (Figure 9.9). This nerve then divides into superficial and deep peroneal nerves. The superficial peroneal nerve innervates the peroneus longus and brevis, as well as the skin in the lower two-thirds of the lateral aspect of the leg and dorsum of the foot. The deep peroneal innervates the tibialis anterior, extensor hallucis, extensor digitorum longus and brevis, and peroneus tertius. The sensory territory of the deep peroneal nerve is the web space between the first and second toes.

Foot drop can be seen with common or deep peroneal lesions, as well as with sciatic nerve lesions, L5 radiculopathies, and plexopathies. Localization is dependent on the distribution of motor abnormalities seen throughout the leg, especially of muscles not innervated by the deep peroneal nerve. In this case, the patient has abnormalities in both deep (tibialis anterior, extensor hallucis, and extensor digitorum brevis) and superficial peroneal nerve–innervated muscles (peroneus longus). Clinically, foot dorsiflexion weakness is attributed to the deep peroneal nerve–innervated muscles, whereas foot eversion weakness is attributed to superficial peroneal nerve–innervated muscles. Therefore, presence of weak dorsiflexion and eversion points toward a common peroneal nerve injury (as opposed to isolated superficial or deep peroneal nerve injuries) like in this patient. The fact that the short head of the biceps femoris is spared suggests that the lesion is distal to this level. The lack of tibial nerve involvement with abnormalities restricted to the peroneal nerve only makes sciatic neuropathy, tibial neuropathy, or L5 radiculopathy less likely. Sciatic nerve injury is discussed in question 32. L5 radiculopathy is discussed in question 86.

Tibial nerve injury is discussed in question 34.

Katirji B. Electromyography in Clinical Practice: A Case Study Approach, 1st ed. St. Louis: Mosby; 1998.

Russell SM. Examination of Peripheral Nerve Injuries. New York: Thieme; 2006.

45. b

The history and examination depicted in question 45 are consistent with mild ulnar neuropathy at the elbow. The ulnar nerve is most susceptible to entrapment at the post-condylar groove. A truck driver who leans his arm against the window sill for prolonged periods of time while driving would be at risk for this type of compression. The symptoms and examination are not consistent with pathology in the median nerve. Similarly, the C6 and C7 myotomes would not be affected by this type of nerve injury, and there are not other historical points suggestive of a radiculopathy. This type of compression is best managed conservatively with avoidance of pressure on the elbow and avoidance of repetitive flexion and extension of the forearm. Surgery is indicated only in select severe cases.

Bradley WG, Daroff RB, Fenichel GM, et al. Neurology in Clinical Practice, 5th ed. Philadelphia, PA: Elsevier; 2008.

Russell SM. Examination of Peripheral Nerve Injuries. New York: Thieme; 2006.

46. e

Diabetic neuropathy has significant clinical implications. Patients with polyneuropathy are at risk for developing foot ulcers and ankle arthropathy (discussed also in Chapter 16).

Foot ulcers are a severe complication caused by a combination of factors, including loss of sensation from neuropathy leading to unnoticed trauma, poor vascular perfusion, and higher risk of infection.

Ankle arthropathy, known as “Charcot joint,” is a joint deformity associated with loss of sensation and sensory ataxia resulting from neuropathy.

Mononeuropathies can involve peripheral or cranial nerves. A typical cranial nerve affected by diabetes is the third nerve, which can occur acutely, usually sparing the pupillary function (discussed in Chapter 1).

Bradley WG, Daroff RB, Fenichel GM, et al. Neurology in Clinical Practice, 5th ed. Philadelphia, PA: Elsevier; 2008.

Ropper AH, Samuels MA. Adams and Victor’s Principles of Neurology, 9th ed. New York: McGraw-Hill; 2009.

47. a

This patient has meralgia paresthetica, which is caused by a mononeuropathy of the lateral femoral cutaneous nerve of the thigh. This nerve originates from L2 and L3, and is purely sensory. Patients with meralgia paresthetica complain of numbness, pain, and paresthesias on the anterolateral aspect of the thigh. Pain may be worse with standing and walking, and better with flexion at the hip or sitting. This condition results from compression of or trauma to this nerve. Predisposing factors include obesity, pregnancy, ascites, devices that compress the nerve at the waist (tight belts), diabetes, and rapid and significant weight loss.

Ropper AH, Samuels MA. Adams and Victor’s Principles of Neurology, 9th ed. New York: McGraw-Hill; 2009.

Russell SM. Examination of Peripheral Nerve Injuries. New York: Thieme; 2006.

48. b

On the basis of the clinical and electrophysiologic findings, this patient has a femoral nerve injury that occurred in the intrapelvic region. Femoral nerve injury in the intrapelvic region can be caused by pelvic surgery, pelvic masses, or retroperitoneal hematomas. The history suggests a retroperitoneal hematoma, which can compress the femoral nerve in the intrapelvic region. This patient has pain with hip flexion and knee extension weakness, as well as absent patellar reflex. There are also sensory findings in the femoral nerve and saphenous nerve distribution. The presence of fibrillations in the iliacus and psoas muscles suggests an intrapelvic injury rather than an inguinal injury, given that these muscles are innervated by the femoral nerve in the intrapelvic region and prior to its course through the inguinal region. The anatomy of the femoral nerve is discussed in question 29 and shown in Figure 9.7.

Involvement of the psoas distinguishes intrapelvic injury from injury at the inguinal ligament (discussed also in question 29). The fact that thigh adductors and other muscle groups are spared suggests that this is not a plexopathy or radiculopathy as only the femoral nerve–innervated muscles are involved. The abnormal SNAPs point away from the diagnosis of a radiculopathy.

Katirji B. Electromyography in Clinical Practice: A Case Study Approach, 1st ed. St. Louis: Mosby; 1998.

Russell SM. Examination of Peripheral Nerve Injuries. New York: Thieme; 2006.

49. a

This patient has a small fiber diabetic neuropathy, which is characterized by painful paresthesias, burning sensations, and allodynia that typically affect both feet, but may affect other regions. There is preservation of vibratory sense and proprioception, as well as deep tendon reflexes. Strength is usually preserved as well. Small fiber neuropathy is commonly associated with autonomic neuropathy, and therefore these patients may have features of autonomic dysfunction.

Diabetic mononeuropathy can occur, affecting either the peripheral or cranial nerves, and usually involves one nerve. If several nerves are involved, the presentation is usually asymmetric, unlike in this case. Large fiber diabetic neuropathy is discussed in question 85. Diabetic polyradiculoneuropathy and diabetic amyotrophy are discussed in question 42.

Bradley WG, Daroff RB, Fenichel GM, et al. Neurology in Clinical Practice, 5th ed. Philadelphia, PA: Elsevier; 2008.

Ropper AH, Samuels MA. Adams and Victor’s Principles of Neurology, 9th ed. New York: McGraw-Hill; 2009.

50. c

This patient’s history and examination are consistent with posterior interosseus nerve palsy, which can occur as a diabetic mononeuropathy, but can also occur in the setting of posterior interosseus nerve compression (such as due to lipomas or nerve sheath tumors) or can be seen in Parsonage-Turner syndrome (discussed in question 52). The posterior interosseus nerve is a pure motor nerve. With a posterior interosseus nerve palsy, there is not an obvious wrist drop because radial nerve branches to extensor carpi radialis longus and brevis originate proximal to the posterior interosseus nerve; there is however weakness of wrist extension in an ulnar direction (see Table 9.6for muscles innervated by the posterior interosseus nerve).

Strength of more proximal radial nerve–innervated muscles excludes a radial neuropathy at the spiral groove or elbow, and intact superficial sensory radial nerve responses on NCS further support this. Intact triceps reflex and normal strength of forearm extension exclude a C7 radiculopathy. Supinator syndrome causes a painful posterior interosseus nerve palsy due to compression or irritation of this nerve as it passes through the supinator muscle. Absence of pain on forced supination makes this uncommon disorder less likely.

Russell SM. Examination of Peripheral Nerve Injuries. New York: Thieme; 2006.

51. b

This patient has an L2-L3-L4 radiculopathy. Radiculopathies involving the upper lumbar roots are more difficult to assess and less common than those involving the lower lumbosacral roots. L2-L3 radiculopathy manifests with pain in the hip and groin radiating down the anterior and medial thigh. If there is an L4 radiculopathy, the pain may also radiate down to the medial leg. L2-L3-L4 radiculopathy may affect hip flexion and knee extension, as well as ankle dorsiflexion due to involvement of the L4 root, which partially innervates the tibialis anterior along with L5. The patellar reflex is reduced or absent in L2-L3-L4 root lesions.

The iliacus muscle is innervated by L2-L3 roots. The quadriceps femoris is innervated by L3-L4 roots, and the tibialis anterior is innervated by L4-L5 roots.

L2-L3-L4 radiculopathies should be distinguished from a lumbar plexopathy and femoral neuropathy. The differences can be difficult to detect clinically, but may be further clarified with EMG. In radiculopathies, the SNAPs are normal, whereas in plexopathies, they are abnormal. Paraspinal fibrillations are seen in radiculopathies, but not in plexopathies. In femoral neuropathy, the manifestations should be restricted to the distribution and muscles supplied by this nerve. In this case, it should also be considered in the differential diagnosis. However femoral nerve injury does not involve the tibialis anterior muscle, which is an L4, peroneal nerve–innervated muscle.

L5 radiculopathy is discussed in question 86. S1 radiculopathy is discussed in question 40. Femoral neuropathy is discussed in questions 29 and 48.

Katirji B. Electromyography in Clinical Practice: A Case Study Approach, 1st ed. St. Louis: Mosby; 1998.

Russell SM. Examination of Peripheral Nerve Injuries. New York: Thieme; 2006.

52. b

The patient has a history and examination consistent with acute brachial plexitis, also known as neuralgic amyotrophy or Parsonage-Turner syndrome. This can occur following surgery, vaccination, or systemic viral illness; in some cases, it can be idiopathic. Symptoms of acute brachial neuritis include acute onset of severe shoulder and arm pain, which then resolve, with subsequent occurrence of weakness. This disorder can potentially affect any portion of the brachial plexus and can be isolated to a single upper extremity nerve, but frequently affects multiple nerves originating from the brachial plexus in a multifocal fashion. Nerves frequently affected include the suprascapular nerve, long thoracic nerve, and median nerve. It can also occur in the lumbosacral plexus. This disorder is most often monophasic with good recovery, but recurrent episodes may occur. There is a familial form, hereditary neuralgic amyotrophy, which is autosomal dominant in inheritance. Recovery typically occurs after each attack, though with time residual deficits may be incurred. Some cases of hereditary neuralgic amyotrophy have been linked to SEPT9 gene on chromosome 17.

The fact that this patient had normal strength postoperatively argues against a nerve injury related to nerve traction, median sternotomy, or other procedure-related etiologies. Delayed weakness postoperatively should also place brachial plexus compression from a hematoma on the differential diagnosis. Infectious polyradiculitis would be unusual in this postoperative setting, and more often occurs in immunocompromised patients, such as cytomegalovirus polyradiculitis in HIV patients. The cervical-brachial-pharyngeal variant of Guillain-Barre is associated with neck and pharyngeal weakness as well, and the occurrence of pain which then resolves and is followed by weakness is more typical of acute brachial plexitis.

Bradley WG, Daroff RB, Fenichel GM, et al. Neurology in Clinical Practice, 5th ed. Philadelphia, PA: Elsevier; 2008.

Ropper AH, Samuels MA. Adams and Victor’s Principles of Neurology, 9th ed. New York: McGraw-Hill; 2009.

53. a, 54. c

The patient in question 53 has a complete median nerve palsy at the level of the antecubital fossa. Brachial artery injury often occurs concomitantly, given its proximity to the median nerve in this area. Intact strength of the ulnar nerve–innervated muscles (see Table 9.5) is evidence that this is not a medial cord lesion. Ischemic monomelia, as can occur during placement of arteriovenous shunts for dialysis, is painful and causes circumferential sensory loss in multiple nerve distributions, a clinical picture not evident in this case. The sensory fibers of the median nerve are derived mainly from C6 and C7 through the lateral cord. Weakness of C6- and C7-innervated muscles, such as the pronators, is evidence that this is not a medial cord lesion because the medial cord carries predominantly C8 and T1 fibers. With complete median nerve palsy, on attempt to make a fist, the first digit does not flex, the second digit partially flexes, and the fourth and fifth digits flex normally, assuming a hand position similar to that used in religious blessings, termed the Benedictine sign.

The patient in question 54 has anterior interosseus nerve syndrome. The anterior interosseus nerve is a pure motor branch of the median nerve and innervates flexor digitorum profundus to the second and third digits, flexor pollicis longus, and pronator quadratus (Table 9.1). Weakness of these muscles, in the absence of sensory loss, suggests isolated involvement of the anterior interosseus nerve, as can occur with trauma, fracture, or in neuralgic amyotrophy (Parsonage-Turner syndrome; see question 52). Patients complain of weakness in grasping objects with their thumb and index finger, and on attempt to make an “okay sign,” the distal phalanges are unable to flex, and instead, the fingertips touch.

In both cases, weakness of C8 and T1 median nerve–innervated muscles is evidence that this is not isolated to the C7 nerve root (see question 23 and Table 9.1 for more information on the course of the median nerve and action of median nerve–innervated muscles).

Bradley WG, Daroff RB, Fenichel GM, et al. Neurology in Clinical Practice, 5th ed. Philadelphia, PA: Elsevier; 2008.

Russell SM. Examination of Peripheral Nerve Injuries. New York: Thieme; 2006.

55. c, 56. c

In patients with Guillain-Barre syndrome (GBS), CSF typically shows albuminocytologic dissociation, which is the presence of increased proteins with low cell counts. The other options in question 55 are not CSF findings to be expected in patients with GBS.

Neurophysiologic studies should be performed early and repeated in 3 weeks if clinically indicated. The main findings are consistent with demyelination, and typically show prolonged or abnormal distal latencies and slow conduction velocities, with abnormal late responses, which are the F-response and the H-reflex. Although distal latencies may not be affected initially, they are almost always affected at some time in the disease process. There may also be evidence of motor conduction block.

Neurophysiologic data can help to evaluate GBS and determine the presence of other variants. The three possible variant subtypes are the following:

– Acute inflammatory demyelinating polyneuropathy: findings of demyelination, with prolonged latencies and slow conduction velocities.
– Acute motor and sensory axonal neuropathy: no features of demyelination, but reduced sensory and motor amplitudes.
– Acute motor axonal neuropathy: no features of demyelination, but reduced motor amplitudes.

MRI of the lumbar spine may show enhancement of the cauda equina.

Hughes RAC, Cornblath DR. Guillain-Barre syndrome. Lancet. 2005; 366:1653–1666.

Ropper AH, Samuels MA. Adams and Victor’s Principles of Neurology, 9th ed. New York: McGraw-Hill; 2009.

57. c

This patient’s history and examination are consistent with radiation-induced brachial plexopathy. Her examination suggests a pan-plexopathy. Radiation-induced plexopathy can occur months to years following exposure to radiation. Unlike carcinomatous invasion, radiation-induced plexopathy is painless. Myokymia on EMG further supports the diagnosis and distinguishes it from carcinomatous invasion. A paraneoplastic neuropathy as well as chemotherapy-induced neuropathy would more likely affect other limbs as well. Cervical spine stenosis would likely affect both limbs (though this is often asymmetric) and would also likely be associated with neck pain as well.

Russell SM. Examination of Peripheral Nerve Injuries. New York: Thieme; 2006.

58. d

This patient has a Miller-Fisher syndrome, which is associated with the presence of the antibody GQ1b. Miller-Fisher syndrome, described by C. Miller Fisher, is a variant of Guillain-Barre syndrome (GBS), characterized by the triad of ataxia, ophthalmoplegia, and areflexia. Antibodies to GQ1b are present in 90% of patients with Miller-Fisher syndrome. Antibodies to GQ1b can also lead to isolated ocular nerve abnormalities.

Other variants of GBS have been associated with antibodies to gangliosides. Gangliosides are glycosphingolipids with sugar residues on the extracellular surface bearing sialic acid molecules. Antibodies to GM1, GM1b, and GD1a have been detected in acute motor axonal neuropathy and acute motor and sensory axonal neuropathy. Antibodies to GalNAc-GD1a can be seen in patients with acute motor axonal neuropathy. Antibodies to GD1b can be seen in acute sensory neuronopathy.

Hughes RAC, Cornblath DR. Guillain-Barre syndrome. Lancet. 2005; 366:1653–1666.

Lee SH, Lim GH, Kim JS, et al. Acute ophthalmoplegia (without ataxia) associated with anti-GQ1b antibody. Neurology. 2008; 71:426–429.

59. b

Pronator teres syndrome results from compression of the median nerve as it passes between the two heads of pronator teres. It is uncommon, but occurs in people who perform repetitive forceful pronation and may be associated with medial epicondylitis, or “golfer’s elbow.” Symptoms include gradual onset of a deep ache in the forearm that may worsen with pronation and weakness in median nerve–innervated muscles. Because branches from the median nerve that innervate the pronator teres arise proximal to this muscle (before the nerve passes under this muscle), pronator teres strength is intact in this syndrome. Apparently normal strength in pronator teres excludes a complete median nerve palsy at the elbow. Weakness of muscles not innervated by the anterior interosseus nerve (such as finger flexors and thumb abductors) indicates that this is not isolated anterior interosseus nerve syndrome. Weakness of flexor carpi radialis, which is predominantly C6 and C7 in innervation (see Table 9.1), indicates that this is not a medial cord lesion.

Bradley WG, Daroff RB, Fenichel GM, et al. Neurology in Clinical Practice, 5th ed. Philadelphia, PA: Elsevier; 2008.

Russell SM. Examination of Peripheral Nerve Injuries. New York: Thieme; 2006.

60. d

Patients with carpal tunnel syndrome often have a history of repetitive motions at the wrist, as occurs in certain occupations. Some systemic illnesses predispose to carpal tunnel syndrome, including diabetes, rheumatoid arthritis, chronic renal failure, amyloidosis, acromegaly, obesity, and hypothyroidism. Testing for these conditions in the appropriate clinical setting may be indicated, especially in the absence of evident occupational risk factors or when the presentation is bilateral. Local compression by masses, such as ganglion cysts or neurofibromas, can cause median nerve compression as well. For mild carpal tunnel syndrome, in which sensory symptoms are intermittent and there is no evidence of axon loss on EMG, management includes wrist splinting, particularly at night, and minimization of provocative maneuvers. Surgical therapy is not indicated for asymptomatic cases, though measures to minimize further median nerve compression at the wrist are indicated. In more severe cases, surgery is indicated when there is significant sensory loss, motor weakness, and/or evidence of significant axon loss. Surgery may be indicated for milder cases if symptoms are significant and refractory to splinting and/or if the patient cannot avoid the provocative actions (due to, for example, employment).

Preston DC, Shapiro BE. Electromyography and Neuromuscular Disorders, 2nd ed. Philadelphia, PA: Elsevier; 2005.

Russell SM. Examination of Peripheral Nerve Injuries. New York: Thieme; 2006.

61. b, 62. d

The patient in question 61 has a history and examination consistent with a radial neuropathy at the axilla. Loss of sensation over the posterior arm (due to involvement of the posterior cutaneous nerve of the arm, the most proximal branch of the radial nerve), as well as weakness of the triceps (which receives innervation from the radial nerve proximal to the spiral groove), indicates that the lesion is proximal to the spiral groove. Proximal radial nerve lesions are not common, but can occur with repetitive pressure to the axilla, as occurs with prolonged crutch use, or with prolonged pressure otherwise on the axilla, as occurs in “Saturday night palsy,” when a person sleeps with his or her arm hung over the back of a chair. Although the triceps reflex (C7) is absent, intact forearm pronation suggests that pronator teres, which is predominantly innervated by the C7 nerve root, is normal, excluding a C7 radiculopathy. Intact arm abduction, mediated in large part by axillary nerve–innervated muscles, excludes a posterior cord lesion. With posterior cord lesions, muscles innervated by both the axillary and radial nerves are weak.

The patient in question 62 has a history and examination consistent with a radial neuropathy at the spiral groove. Intact strength of the triceps and intact sensation on the posterior aspect of the arm support that the lesion is distal to origin of the posterior cutaneous nerve to the arm and branches to triceps, excluding a proximal radial nerve lesion or a posterior cord lesion; intact triceps strength and normal triceps deep tendon reflex are evidence that this not a C7 radiculopathy. Reduced sensation over the lateral aspect of the arm suggests that the lesion is more proximal than the elbow, as the lateral cutaneous nerve to the arm arises from the radial nerve within the spiral groove. The spiral groove is the most common location for radial nerve injury, and a common mechanism is humeral fracture. Other choices are excluded on the basis of the discussion above.

Russell SM. Examination of Peripheral Nerve Injuries. New York: Thieme; 2006.

63. d

This man’s history and examination are consistent with injury to the spinal accessory nerve, which is rare in isolation, but can be an iatrogenic complication of radical neck dissection, because in the neck, it is interwoven among several lymph nodes. The spinal accessory nerve carries fibers from the lower medulla as well as C1 to C4. It exits the skull base through the jugular foramen and innervates the sternocleidomastoid, which turns the head contralaterally, as well as the trapezius, which is involved in shoulder shrug along with assisting in elevation of the scapula and assisting the deltoid in arm abduction beyond 90 degrees.

Jugular foramen syndrome, or Vernet’s syndrome, results from compressive lesions of the foramen magnum, such as metastases or schwannomas, and there would be evidence of involvement of the vagus and glossopharyngeal nerves in addition to the spinal accessory nerve.

The cervical plexus consists of the ventral rami of C1 to C4. The cervical plexus provides innervation to several neck muscles including the levator scapula (C3 and C4), which significantly contributes to shoulder shrug (along with trapezius), allowing relatively preserved shoulder shrug strength in the setting of trapezial weakness. The cervical plexus is also the origin of the nerves that supply sensory innervation to the posterior head and neck regions, including the lesser occipital nerves that provide sensation to the area posterior to the ear, the greater occipital nerves, which carry predominantly C2 fibers and provide sensory innervation to most of the posterior head, the greater auricular nerves, and the transverse cervical nerves. Absence of sensory loss in this patient excludes the other choices listed.

Russell SM. Examination of Peripheral Nerve Injuries. New York: Thieme; 2006.

64. e

Diabetes mellitus involvement of the peripheral nervous system can present in various forms, with a distal symmetric polyneuropathy being the most common presentation. This polyneuropathy is commonly a large fiber length-dependent neuropathy, but small fiber neuropathy is also seen (discussed in question 49). Other presentations include autonomic neuropathy (discussed in question 20), polyradiculopathy (diabetic amyotrophy; discussed in question 42), mononeuropathy affecting either peripheral or cranial nerves, and multiple mononeuropathies.

Bradley WG, Daroff RB, Fenichel GM, et al. Neurology in Clinical Practice, 5th ed. Philadelphia, PA: Elsevier; 2008.

65. d

This patient has multifocal acquired demyelinating sensory and motor neuropathy(MADSAM). This is a demyelinating neuropathy with evidence of conduction block, presenting with asymmetric motor and sensory symptoms. The progression is gradual and slow, usually involving the upper limbs initially and later the lower limbs. Deep tendon reflexes are diminished in the distribution of the affected nerves. On CSF examination, the protein is usually elevated. Unlike multifocal motor neuropathy (discussed in questions 35 and 36), anti-GM1 antibodies are not present, and electrophysiologic studies show abnormalities in motor and sensory nerves, with features of demyelination and conduction block. Patients with MADSAM may improve on steroids.

Chronic inflammatory demyelinating polyneuropathy (CIDP) is discussed in questions 11, 12, and 38. Acute inflammatory demyelinating polyneuropathy (AIDP) is discussed in questions 15, 16, 24, 55, and 56. Subacute inflammatory demyelinating polyneuropathy is a controversial diagnosis that is not generally accepted, but some authors have used this term to define patients with manifestations similar to AIDP and CIDP, with a time evolution between 4 and 8 weeks.

Saperstein DS. Chronic acquired demyelinating polyneuropathies. Semin Neurol. 2008; 28:168–184.

66. d

This patient’s history and examination are consistent with a superficial sensory radial neuropathy. This type of neuropathy, also called Wartenberg’s syndrome, can result from compression or irritation of this nerve due to tight handcuffs or watches, venipuncture, or surgery. Rarely, compression may occur due to pinching of the nerve between the brachioradialis and extensor carpi radialis longus tendons as occurs with repetitive pronation. Symptoms include dysesthesias and numbness over the dorsolateral aspect of the hand. There is no motor weakness, as the superficial sensory radial nerve is a pure sensory nerve. Treatment is conservative, and includes avoidance of pressure to the nerve and medications for neuropathic pain (such as amitriptyline, pregabalin, or gabapentin) if necessary.

Russell SM. Examination of Peripheral Nerve Injuries. New York: Thieme; 2006.

67. e

This patient’s history and examination are consistent with a musculocutaneous neuropathy. Such neuropathies in isolation are rare, but can occur with anterior shoulder dislocations and other types of trauma. The musculocutaneous nerve is a continuation of the lateral cord and carries predominantly C5 and C6 fibers. The musculocutaneous nerve innervates the coracobrachialis muscle that assists the deltoid in anterior flexion of the arm at the shoulder and stabilizes the humerus during forearm flexion. The musculocutaneous nerve then innervates the brachialis muscle and the biceps brachii, which flex the forearm at the elbow. The brachioradialis, innervated by the radial nerve, also contributes to forearm flexion. The biceps brachii also supinates the forearm, and is the main forearm supinator when the forearm is flexed.

The musculocutaneous nerve provides sensory innervation to the lateral half of the forearm via the lateral antebrachial cutaneous nerve, but this nerve does not provide any sensation below the wrist. The latter point, along with intact strength of the brachioradialis in the case (as evidence by stronger forearm supination with the forearm extended), is evidence that this is not a C6 radiculopathy. Absence of palpable contraction of the biceps during attempted forearm flexion is evidence that this is not biceps tendon rupture. A lateral cord lesion would lead to weakness in other C5- to C7-innervated muscles, such as flexor carpi radialis and pronator teres (see Tables 9.1 and 9.6). It would also lead to sensory loss over the palm, palmar aspect of the thumb, and second and third digits. Therefore, absence of weakness in wrist flexion and forearm pronation distinguishes a lateral cord lesion from musculocutaneous palsy. A median neuropathy would lead to weakness in muscles innervated by the median nerve (see Table 9.1), not evident in this case.

Russell SM. Examination of Peripheral Nerve Injuries. New York: Thieme; 2006.

68. e

EMG/NCS are normal in patients with pure small fiber neuropathy.

Small fibers include myelinated A-δ and unmyelinated C-fibers, which are involved in autonomic, temperature, and pain transmission. Patients with small fiber neuropathy present with painful burning sensations and dysesthesias distally (most frequently in the feet). Some patients may have autonomic manifestations that compromise sweating, vasomotor control, gastrointestinal, and genitourinary functions. In pure small-fiber neuropathy, besides sensory findings, the neurologic examination is normal, including motor and reflex examination, distinguishing it from large-fiber neuropathy.

A cause is not found in the majority of cases. There are multiple causes of small-fiber neuropathy that account for a small percentage of the cases and should be investigated. Of the cases in which an etiology is found, diabetes and impaired glucose tolerance are the most common. Others include alcohol, amyloidosis, vasculitis, sarcoidosis, HIV, hyperlipidemia, Sjogren’s syndrome, and connective tissue disorders.

EMG/NCS are normal because these tests evaluate the integrity of large nerve fibers. Tests that help in making the diagnosis include quantitative sudomotor axon reflex test (QSART), thermoregulatory sweat test (TST) and skin biopsy (discussed also in Chapter 10). QSART evaluates postganglionic sympathetic cholinergic sudomotor function, and is performed by stimulation of sweat glands by iontophoresis of acetylcholine. TST assesses the pattern of sweating and dysfunction of sweating by placing the patient in a warming chamber while covered by a reactive powder that changes color with sweat. Skin biopsy determines if intraepidermal nerve fiber density is significantly reduced.

The treatment should target the underlying cause when determined. Symptomatic treatment includes medications for neuropathic pain, including gabapentin, pregabalin, carbamazepine, and amitriptyline, among others.

Al-Shekhlee A, Chelimsky TC, Preston DC. Small-fiber neuropathy. Neurologist. 2002; 8:237–253.

Hoitsma E, Reulen JPH, de Baets M, et al. Small fiber neuropathy: A common and important clinical disorder. J Neurological Sci. 2004; 227:119–130.

69. a

This patient’s history and examination are consistent with an axillary neuropathy. Axillary neuropathies occur with fractures at the surgical neck of the humerus and with anterior shoulder dislocations. The axillary nerve is a continuation of the posterior cord, and carries predominantly C5 and C6 fibers. The axillary nerve innervates the deltoid muscle, which is the main arm abductor, particularly between 30 and 90 degrees (supraspinatus significantly contributes to arm abduction in the first 30 degrees of abduction and the trapezius contributes to greater than 90 degrees). The deltoid has three heads: (1) the anterior head, which is involved arm flexion (in front of the body), assisted by serratus anterior, (2) the lateral head, which along with the anterior head is mainly involved in arm abduction to the side and slightly anteriorly, and (3) the posterior head, which is involved in posterior movement of the abducted arm. The axillary nerve also innervates teres minor, which externally rotates the arm along with infraspinatus. The axillary nerve provides sensory innervation to the upper lateral arm through the upper lateral brachial cutaneous nerve.

Intact arm abduction in the first 30 degrees suggests the supraspinatus is of normal strength. In addition, there is intact sensation over the posterolateral arm, territory innervated by the C5 nerve root through the lateral and posterior cutaneous nerves to the arm (radial nerve branches). These findings make a C5 radiculopathy unlikely.

Intact strength of radial nerve–innervated muscles excludes a posterior cord lesion and a radial neuropathy; intact strength of predominantly C7-innervated muscles (see Tables 9.1 and 9.6) excludes a middle trunk lesion.

Russell SM. Examination of Peripheral Nerve Injuries. New York: Thieme; 2006.

70. b

This patient’s history is consistent with Fabry’s disease, an X-linked disorder that results from a deficiency in the enzyme α-galactosidase A, a lysosomal enzyme. Females can be affected, but the phenotype is milder than in males, with renal failure being less common. It is a lysosomal storage disease that results from accumulation of globotriaosylceramide in various organs. Commonly involved organs include the kidneys, heart, and skin. Typical skin findings include angiokeratomas, dark punctuate lesions that are often found on the trunk, and intertriginous areas such as the axilla and scrotum. Both central and peripheral nervous system complications can occur. Peripheral nervous system complications include small fiber neuropathy and autonomic neuropathy. Strokes in Fabry’s disease can be cardioembolic or due to large or small vessel occlusion. Endothelial deposition of globotriaosylceramide can lead to dolichoectasia increasing risk for thrombosis. Enzyme replacement therapy for this disorder is available.

Deficiency of arylsulfatase A leads to metachromatic leukodystrophy (discussed in Chapter 14).

Bradley WG, Daroff RB, Fenichel GM, et al. Neurology in Clinical Practice, 5th ed. Philadelphia, PA: Elsevier; 2008.

Testaj FD, Gorelick PB. Inherited metabolic disorders and stroke part 1: Fabry disease and mitochondrial myopathy, encephalopathy, lactic acidosis and strokelike episodes. Arch Neurol. 2010; 67:19–24.

71. c

The distribution of muscle weakness and sensory loss, combined with the EMG findings, suggests a proximal lesion at the C5 and C6 root level. The key clue to this localization is involvement of the rhomboids, which are innervated by the dorsal scapular nerve, which arises from the C5 nerve root. With chronic denervation of the rhomboids, intrascapular wasting can occur. It is important to note that C5 has a significant contribution to the phrenic nerve, which also receives contributions from C3 and C4. A proximal lesion to C5 can lead to weakness of the ipsilateral diaphragm.

Bradley WG, Daroff RB, Fenichel GM, et al. Neurology in Clinical Practice, 5th ed. Philadelphia, PA: Elsevier; 2008.

Preston DC, Shapiro BE. Electromyography and Neuromuscular Disorders, 2nd ed. Philadelphia, PA: Elsevier; 2005.

72. b

The history and examination are consistent with suprascapular nerve entrapment, as can occur in athletes, but also due to trauma (shoulder dislocation or scapular fracture). The clinical picture is one of poorly localizable shoulder pain, and weakness of the supraspinatus, which abducts the arm, particularly during the first 30 degrees of abduction, and infraspinatus, which externally rotates the shoulder when the elbow is flexed and fixed at the patient’s side.

The distribution of weakness in this patient is not consistent with long thoracic nerve injury. The long thoracic nerve arises from the C5, C6, and C7 roots and innervates serratus anterior, which acts to abduct the scapula. Injury to it causes winging of the scapula, which is most evident when the arms are extended and pressure is applied anteriorly (as if doing a push-up on a wall). Scapular winging also occurs with rhomboid weakness. Normal strength of forearm flexion suggests the biceps, a largely C5-innervated muscle, is normal; an intact biceps deep tendon reflex is further evidence that this is not a C5 radiculopathy. For similar reasons, an upper trunk lesion is also excluded.

The history is also not consistent with thoracodorsal nerve injury because arm adduction is normal in this patient; the thoracodorsal nerve arises from the posterior cord and innervates the latissimus dorsi, which acts to adduct the arm. The other two branches of the posterior cord are also involved in arm abduction (as well as internal rotation): (1) the upper subscapular nerve, which innervates the subscapularis muscle, and (2) the lower subscapular nerve, which innervates teres major as well as a portion of the subscapularis muscle.

Russell SM. Examination of Peripheral Nerve Injuries. New York: Thieme; 2006.

73. d

This patient has a thoracoabdominal polyradiculopathy, which is seen in patients with long-standing diabetes, and presents with pain and dysesthesias, patchy sensory and motor changes in thoracic and abdominal nerve root territories, usually unilateral but may be bilateral. It is commonly confused with intra-abdominal processes and extensive gastrointestinal work-ups are often undertaken before the diagnosis is made. The pathology is not known but thought to be an ischemic radiculopathy. EMG of the abdominal wall and paraspinals may assist with the diagnosis, showing fibrillations in the involved muscles in one or more adjacent myotomes. Recovery is protracted, and may occur spontaneously or with the treatment of the diabetes.

The clinical presentation in this patient is not consistent with spinal cord ischemia or a femoral neuropathy. The other options are not likely in this case.

Boulton AJ, Angus E, Ayyar DR, et al. Diabetic thoracic polyradiculopathy presenting as abdominal swelling. Br Med J (Clin Res Ed). 1984; 289:798–799.

Ropper AH, Samuels MA. Adams and Victor’s Principles of Neurology, 9th ed. New York: McGraw-Hill; 2009.

74. c

This patient’s history and physical examination are consistent with neurogenic thoracic outlet syndrome. The signs and symptoms result from compression on the C8 and T1 nerve roots. The brachial plexus passes through the scalene triangle, which is formed by the anterior scalene, middle scalene, and first rib. An anomalous fibrous band between the scalene muscles, a cervical rib, or an elongated C7 transverse process can lead to neural compression or irritation, resulting in neurogenic thoracic outlet syndrome. There is weakness of intrinsic hand muscles and sensory loss in a C8 and T1 distribution; the pattern of weakness distinguishes this disorder from the other disorders listed, which would lead to weakness in muscle groups innervated by the single nerves listed. Arm abduction and external rotation can precipitate symptoms and reduce the radial pulse. Sensory loss in both C8 and T1 dermatomes distinguishes thoracic outlet syndrome from C8 radiculopathy. Involvement of the medial antebrachial cutaneous nerve makes thoracic outlet syndrome more likely than ulnar neuropathy at the elbow.

Russell SM. Examination of Peripheral Nerve Injuries. New York: Thieme; 2006.

75. a, 76. d

77. e

The history and examination of the patient in question 75 suggest the diagnosis of hereditary sensory and autonomic neuropathy (HSAN) type 1. In question 76, the diagnosis is HSAN3. The patient in question 77 suffers from HSAN4.

The HSANs are a relatively rare, genetically and phenotypically heterogeneous group of hereditary neuropathies. As a group, they share in common prominent sensory signs and symptoms, including pain, sensory loss, and autonomic features with little motor involvement. Because of the sensory loss, patients with HSANs are prone to painful calluses, stress fractures, neuropathic (Charcot) joints, skin ulcerations that heal poorly, and infections with deep tissue involvement, such as osteomyelitis, leading to disfiguring acral mutilations.

Question 75 depicts HSAN1. HSAN1 is the most common HSAN. It is autosomal dominant. Presentation is typically in young adulthood. Painful sensory symptoms such as lancinating pains are prominent. There is relatively dissociated sensory loss, with pain and temperature affected more than dorsal column modalities. The main autonomic manifestation is hypohydrosis. Only in advanced cases do muscle weakness and atrophy occur. Hearing loss can rarely occur. HSAN1 results from a mutation in the gene encoding for serine palmitoyltransferase, which catalyzes the rate-limiting step of sphingolipid synthesis.

HSAN2 begins in infancy, and is characterized by generalized loss of sensation and insensitivity to pain, leading to significant risk of mutilation to the hands, feet, lips, and tongue. Autonomic symptoms are not prominent, and cognitive function is normal. Associated features include areflexia and retinitis pigmentosa. NCS show evidence of axon loss, with absence of SNAPs.

Question 76 depicts HSAN3, also known as familial dysautonomia or Riley-Day syndrome, an autosomal recessive HSAN with prominent autonomic features. Symptom onset is in infancy with dysphagia, vomiting, recurrent infections, and blood pressure lability. It is particularly prevalent among Ashkenazi Jews. With emotional stimulation, there is hyperhydrosis, skin flushing, and hypertension. Other autonomic features include absence of lacrimation. The tongue may be smooth due to absence of fungiform papillae. Later in life, evidence of a predominantly sensory neuropathy with insensitivity to pain occurs, as well as areflexia. Nerve biopsy shows a marked reduction in the density of unmyelinated axons and small myelinated axons, with a reduction in autonomic ganglia cell bodies. HSAN3 is due to a mutation in the IKAP gene, which results in abnormal mRNA splicing, leading to dysregulation of neural endocytosis.

Question 77 depicts HSAN4, or congenital insensitivity to pain. This is an autosomal recessive disorder marked by insensitivity to pain, leading to repeated injury and self-mutilation. Cognitive delay is also present, as are significant behavioral problems including hyperactivity. Autonomic features include anhidrosis, leading to heat intolerance and frequent fevers. There is little evidence of large fiber sensory or motor neuropathy on examination or EMG/NCS; diagnosis is made by demonstration of absent or a markedly reduced number of unmyelinated axons and small myelinated fibers on skin biopsy, absence of nerve endings in sweat glands, as well as absence of sweating by quantitative sudomotor axon-reflex test (discussed in Chapter 10). This disorder results from mutations in the tyrosine kinase receptor for nerve growth factor NTRK1, which plays a role in development of unmyelinated nociceptive and sudomotor fibers.

Prominence of sensory features with relatively few motor manifestations and the presence of autonomic signs and symptoms distinguish the HSANs from Charcot-Marie-Tooth (CMT).

CMT1A does not typically include features of early prominent dysphagia or autonomic dysfunction, but other features are present, as discussed in questions 21 and 22.

Bradley WG, Daroff RB, Fenichel GM, et al. Neurology in Clinical Practice, 5th ed. Philadelphia, PA: Elsevier; 2008.

Ropper AH, Samuels MA. Adams and Victor’s Principles of Neurology, 9th ed. New York: McGraw-Hill; 2009.

78. c

The clinical presentation of Guillain-Barre syndrome is a symmetric ascending sensorimotor neuropathy, and not mononeuritis multiplex.

Multiple mononeuropathies or mononeuritis multiplex refers to the involvement of two or more nerves, usually with acute to subacute onset, in which subsequent nerves are involved at irregular intervals. Common causes of mononeuritis multiplex are vasculitic neuropathy, either isolated or caused by systemic conditions such as polyarteritis nodosa, Wegener granulomatosis, Sjogren’s syndrome, or Churg-Strauss syndrome (discussed in Chapter 16). Cryoglobulinemia can also produce this presentation. Infectious processes such as Lyme disease and HIV can also produce mononeuritis multiplex.

Diabetes can present not only with a distal symmetric polyneuropathy, but can also be associated with mononeuritis multiplex. Other causes of mononeuritis multiplex include sarcoidosis, paraneoplastic processes, amyloidosis, leprosy, systemic lupus erythematosus, rheumatoid arthritis, and lymphoma.

Bradley WG, Daroff RB, Fenichel GM, et al. Neurology in Clinical Practice, 5th ed. Philadelphia, PA: Elsevier; 2008.

Ropper AH, Samuels MA. Adams and Victor’s Principles of Neurology, 9th ed. New York: McGraw-Hill; 2009.

79. c

This patient has cryoglobulinemia, in which complement levels are reduced.

The clinical manifestation of cryoglobulinemia includes nonspecific constitutional symptoms, palpable purpura, arthralgias, lymphadenopathy, hepatosplenomegaly, and peripheral neuropathy, including mononeuritis multiplex. Cryoglobulins are immunoglobulins that precipitate when exposed to cold temperatures and redissolve on rewarming (discussed also in Chapter 16). This condition is frequently associated with hepatitis C, and sometimes has been found in patients with HIV. Other associated conditions are the monoclonal gammopathies, such as multiple myeloma, and connective tissue diseases. Treatment includes steroids and cyclophosphamide, and in some cases, plasmapheresis has shown beneficial results.

Ropper AH, Samuels MA. Adams and Victor’s Principles of Neurology, 9th ed. New York: McGraw-Hill; 2009.

80. b

This patient’s history is consistent with primary erythromelalgia, a rare autosomal dominant disorder characterized by episodes of severe burning and erythema of the distal extremities with heat exposure or exercise. Patients are asymptomatic between episodes. This disorder is due to mutations in the voltage-gated sodium channel SCN9A gene, which results in hyperactivity of the dorsal root ganglia.

Secondary erythromelalgia can be seen with polycythemia rubra vera and other myeloproliferative disorders; the family history and normal routine laboratory testing in this patient make primary erythromelalgia the more likely diagnosis. Similarly, there is no evidence of small fiber neuropathy due to glucose intolerance, and the episodic nature of the symptoms further excludes that diagnosis. The episodic nature of the symptoms with normal examination otherwise is evidence that this is not hereditary sensory and autonomic neuropathy type I. Fabry’s disease can lead to painful acroparesthesias that are worsened with heat exposure and exercise, but evidence of small fiber neuropathy on examination and other findings on history and examination distinguish between these two disorders (discussed in question 70).

Bradley WG, Daroff RB, Fenichel GM, et al. Neurology in Clinical Practice, 5th ed. Philadelphia, PA: Elsevier; 2008.

81. e

This patient has a sensory neuronopathy or ganglionopathy, in which the dorsal root ganglia are involved, presenting with progressive sensory deficits that are usually non-length dependent, patchy, and asymmetric, and lead to global sensory loss. Sensory ataxia is a characteristic finding of sensory neuronopathy. Patients are also areflexic; however, strength tends to be normal. Some patients may also have autonomic dysfunction.

Sensory neuronopathy is frequently secondary to a primary pathology, and can be paraneoplastic, especially associated with small cell lung cancer, in which Anti-Hu antibodies are typically present. It may be associated with other neoplasms, such as neuroendocrine tumors, lymphomas, breast and ovarian cancers, and sarcomas. Another condition that typically can cause sensory neuronopathy is Sjogren’s syndrome, which is a condition characterized by inflammation of the exocrine glands leading to keratoconjunctivitis sicca and xerostomia (discussed in Chapter 16). The diagnostic work-up includes Schirmer’s test, SSA and SSB antibodies, and lip biopsy to detect inflammatory changes in small salivary glands. Other causes of sensory neuronopathy include HIV infection, HTLV-1, Epstein-Barr virus, varicella zoster virus, measles, monoclonal gammopathies, nicotinic acid deficiency, vitamin E deficiency, riboflavin deficiency, and drugs such as carboplatin, doxorubicin, suramin, thallium, and penicillin. Pyridoxine intoxication is one of the causes of sensory neuronopathy, and therefore should not be supplemented.

Ropper AH, Samuels MA. Adams and Victor’s Principles of Neurology, 9th ed. New York: McGraw-Hill; 2009.

Sghirlanzoni A, Pareyson D, Lauria G. Sensory neuron diseases. Lancet Neurol. 2005; 4:349–362.

82. d

This patient’s history and examination are consistent with a lower trunk lesion due to stretching, as occurs with excessive arm abduction such as when grabbing onto something during falling, motor vehicle accidents, or less commonly with birth injury. Lower trunk lesions lead to weakness in ulnar and median nerve–innervated muscles (see Table 9.1), leading to weakness of intrinsic hand muscles and sensory loss on the medial forearm and hand. Sensory loss occurs in a C8 and T1 distribution; the medial arm, innervated predominantly by T1 and T2, often has preserved sensation.

In the patient described in question 82, muscle weakness in the distribution of both of the median and ulnar nerves makes ulnar neuropathy and median neuropathy as isolated disorders unlikely; the distribution of weakness is not consistent with a radial neuropathy. A lateral cord lesion would lead to weakness of musculocutaneous-innervated muscles as well as C6- and C7-innervated median nerve muscles (see Table 9.1), which are not involved in this case.

Russell SM. Examination of Peripheral Nerve Injuries. New York: Thieme; 2006.

83. e

This patient’s history is consistent with an acute hepatic porphyria. The acute hepatic porphyrias are autosomal dominant and involve multiple areas of the neuraxis. They result from enzymatic dysfunction in porphyrin metabolism with subsequent abnormalities in heme synthesis and metabolism. These disorders include acute intermittent porphyria (AIP), which results from various mutations that lead to reduced activity of the enzyme porphobilinogen deaminase, variegate porphyria, which results from reduced activity of the enzyme protoporphinogen IX oxidase, hereditary coproporphyria which results from reduced activity in the enzyme coprophorphinogen oxidase, and plumboporphyria, which results from reduced enzymatic activity of δ-aminolevulinic acid (ALA) dehydratase. Because the dysfunctional enzymes have some residual enzyme activity, symptoms often do not appear until adolescence or later, with some symptoms occurring in episodes with exposure to certain triggers. Common triggers include medications, menstruation, and alcohol exposure. With exposure to these triggers, activity of the hepatic enzyme ALA synthase increases, leading to overproduction of heme precursors that cannot be sufficiently metabolized by the involved downstream enzymes.

The different porphyrias differ in several biochemical aspects, but the neurologic manifestations of all four disorders are similar. Symptoms typically manifest in early adulthood, with females being more commonly and more severely affected as compared to males. Symptoms typically begin with abdominal pain and other gastrointestinal symptoms, followed by neurologic symptoms, including most prominently manifestations of autonomic instability (tachycardia, labile hypertension, orthostasis, and urinary retention). Neuropsychiatric symptoms including psychosis occur, and seizures may be present as well. In some patients, a subacute predominantly motor neuropathy occurs; the arms may be affected prior to the legs, and proximal muscles are involved more than distal muscles. Involvement of the radial nerve in isolation may occur. The neuropathy is both axonal and demyelinating. Cranial nerve involvement, and even respiratory muscle involvement leading to respiratory failure, may occur.

In AIP, photosensitivity is not present, whereas with variegate porphyria and hereditary coproporphyria, photosensitivity with skin blistering and hyperpigmentation occur.

During an attack, elevated levels of porphobilinogen and aminolevulinic acid are detectable in the urine and serum. Elevated protoporphyrinogen and coproporphyrinogen in the stool distinguish variegate porphyria and hereditary coproporphyria from AIP.

Management of the porphyrias involves prevention of attacks by avoidance of drugs that may precipitate the attack (including antiepileptics, particularly barbiturates) and other triggers. Treatment of attacks includes supportive care, a high-carbohydrate diet, and in some cases hematin, which suppresses δ-ALA synthase activity.

Although Wilson’s disease is on the differential diagnosis of patients with neuropsychiatric symptoms, it does not typically lead to peripheral motor mononeuropathies or abdominal pain, except in the setting of hepatic impairment, and there is no evidence of liver disease in the case presented (see Chapter 6 for more on Wilson’s disease). The history presented is not consistent with acute disseminated encephalomyelitis (discussed in Chapter 7), in which the MRI would be abnormal, or acute inflammatory demyelinating polyneuropathy (discussed in questions 15, 16, 24, 55, and 56). Although systemic lupus erythematosuis can lead to a similar clinical picture, normal MRI of the brain and CSF analysis make this less likely.

Bradley WG, Daroff RB, Fenichel GM, et al. Neurology in Clinical Practice, 5th ed. Philadelphia, PA: Elsevier; 2008.

Ropper AH, Samuels MA. Adams and Victor’s Principles of Neurology, 9th ed. New York: McGraw-Hill; 2009.

84. b

This patient’s history and examination are consistent with an upper trunk lesion. This type of injury is called Erb’s palsy and commonly occurs when the shoulder is forcefully pulled down while the neck is flexed in the opposite direction, as can occur with birth injury. Other mechanisms of injury include falls and motorcycle accidents. The posture held by the patient depicted in question 84, “waiter’s tip” position, is a classic example of Erb’s palsy. Arm adduction and internal rotation result from unopposed action of the pectoralis major; the clavicular head of the pectoralis major is innervated by the lateral pectoral nerve that arises from the lateral cord and acts to adduct and internally rotate the arm. Forearm extension results from unopposed action of the triceps and forearm pronation from unopposed action of pronator teres; both of these muscles are predominantly C7 innervated, and normal strength in these muscles as well as a normal triceps deep tendon reflex excludes a C7 root lesion and a middle trunk lesion (because the middle trunk carries only C7 fibers).

Normal rhomboids on EMG suggest that the C5 and C6 nerve roots, which are the origin of the dorsal scapular nerves, are normal (discussed in questions 17 and 71). This patient’s deficits, including weakness of wrist flexors, suggest the lesion is more extensive than just an axillary neuropathy.

Bradley WG, Daroff RB, Fenichel GM, et al. Neurology in Clinical Practice, 5th ed. Philadelphia, PA: Elsevier; 2008.

Preston DC, Shapiro BE. Electromyography and Neuromuscular Disorders, 2nd ed. Philadelphia, PA: Elsevier; 2005.

85. b

This patient has a large fiber diabetic neuropathy. This is usually a length-dependent neuropathy, in which the patient has numbness and paresthesias that are painless. The distribution is symmetric, affecting hands and feet in a glove and stocking distribution. On examination, there is loss vibratory sense and proprioception, as well as loss of deep tendon reflexes. Sensory ataxia may occur later in the course of the disease. Weakness may occur, but is not a prominent feature.

Diabetic mononeuropathy can affect either the peripheral or cranial nerves, and usually involves one nerve. If several nerves are involved, the presentation is usually asymmetric, unlike in this case. Small fiber diabetic neuropathy is discussed in question 49. Diabetic polyradiculoneuropathy and diabetic amyotrophy are discussed in question 42.

Bradley WG, Daroff RB, Fenichel GM, et al. Neurology in Clinical Practice, 5th ed. Philadelphia, PA: Elsevier; 2008.

Ropper AH, Samuels MA. Adams and Victor’s Principles of Neurology, 9th ed. New York: McGraw-Hill; 2009.

86. a

L5 radiculopathy may manifest as pain from the buttock radiating down the lateral thigh, anterolateral leg, and dorsum of the foot, with sensory impairment in this dermatomal region extending to the big toe. The weakness is prominent on toe extension and ankle dorsiflexion, as well as inversion and eversion of the foot. The only reflex found to be abnormal is the hamstring reflex, which is not routinely checked. As foot drop is a frequent manifestation that may be seen with both an L5 root lesion and common peroneal neuropathy, one important means of distinguishing the two is seen on NCS. Superficial peroneal SNAPs are abnormal in common peroneal nerve lesions, but normal in L5 radiculopathies. Another key diagnostic feature is to detect abnormalities in L5-innervated muscles that are not innervated by the peroneal nerve, such as the tibialis posterior and flexor digitorum longus, both of which are innervated by the tibial nerve. The L5 nerve root provides innervation to the tensor fascia latae, gluteus medius, semitendinosus and semimembranosus, tibialis anterior, extensor hallucis, peroneus longus, extensor digitorum brevis, tibialis posterior, and flexor digitorum longus muscles (see Tables 9.2 and 9.4).

The distribution of the involvement in this patient does not correlate with an S1 radiculopathy, which is discussed in question 40. Common and deep peroneal neuropathies are discussed in question 44. A lesion affecting these nerves individually is not likely in this case, given the presence of involvement of other nonperoneal L5-innervated muscles.

In this case, the SNAPs are spared, which makes the possibility of a sciatic nerve injury unlikely as opposed to a radiculopathy. Sciatic nerve injury is discussed in question 32.

Katirji B. Electromyography in Clinical Practice: A Case Study Approach, 1st ed. St. Louis: Mosby; 1998.

Russell SM. Examination of Peripheral Nerve Injuries. New York: Thieme; 2006.



If you find an error or have any questions, please email us at admin@doctorlib.org. Thank you!